Adult Health Exam 2

Pataasin ang iyong marka sa homework at exams ngayon gamit ang Quizwiz!

The nurse is caring for an adult patient with HF who is prescribed digoxin. When assessing the patient for adverse effects, the nurse should assess for which of the following signs and symptoms? A) Confusion and bradycardia B) Uncontrolled diuresis and tachycardia C) Numbness and tingling in the extremities D) Chest pain and shortness of breath

A Feedback: A key concern associated with digitalis therapy is digitalis toxicity. Symptoms include anorexia, nausea, visual disturbances, confusion, and bradycardia. The other listed signs and symptoms are not characteristic of digitalis toxicity.

An older adult patient has been diagnosed with aortic regurgitation. What change in blood flow should the nurse expect to see on this patients echocardiogram? A) Blood to flow back from the aorta to the left ventricle B) Obstruction of blood flow from the left ventricle C) Blood to flow back from the left atrium to the left ventricle D) Obstruction of blood from the left atrium to left ventricle

A Feedback: Aortic regurgitation occurs when the aortic valve does not completely close, and blood flows back to the left ventricle from the aorta during diastole. Aortic regurgitation does not cause obstruction of blood flow from the left ventricle, blood to flow back from the left atrium to the left ventricle, or obstruction of blood from the left atrium to left ventricle.

The nurse working on a cardiac care unit is caring for a patient whose stroke volume has increased. The nurse is aware that afterload influences a patients stroke volume. The nurse recognizes that afterload is increased when there is what? A) Arterial vasoconstriction B) Venous vasoconstriction C) Arterial vasodilation D) Venous vasodilation

A Feedback: Arterial vasoconstriction increases the systemic vascular resistance, which increases the afterload. Venous vasoconstriction decreases preload thereby decreasing stroke volume. Venous vasodilation increases preload.

The nurse is caring for a patient who has been diagnosed with an elevated cholesterol level. The nurse is aware that plaque on the inner lumen of arteries is composed chiefly of what? A) Lipids and fibrous tissue B) White blood cells C) Lipoproteins D) High-density cholesterol

A Feedback: As T-lymphocytes and monocytes infiltrate to ingest lipids on the arterial wall and then die, a fibrous tissue develops. This causes plaques to form on the inner lumen of arterial walls. These plaques do not consist of white cells, lipoproteins, or high-density cholesterol.

The nurse is performing an initial assessment of a client diagnosed with HF. The nurse also assesses the patients sensorium and LOC. Why is the assessment of the patients sensorium and LOC important in patients with HF? A) HF ultimately affects oxygen transportation to the brain. B) Patients with HF are susceptible to overstimulation of the sympathetic nervous system. C) Decreased LOC causes an exacerbation of the signs and symptoms of HF. D) The most significant adverse effect of medications used for HF treatment is altered LOC.

A Feedback: As the volume of blood ejected by the heart decreases, so does the amount of oxygen transported to the brain. Sympathetic stimulation is not a primary concern in patients with HF, although it is a possibility. HF affects LOC but the reverse is not usually true. Medications used to treat HF carry many adverse effects, but the most common and significant effects are cardiovascular.

A patient is brought to the ED and determined to be experiencing symptomatic sinus bradycardia. The nurse caring for this patient is aware the medication of choice for treatment of this dysrhythmia is the administration of atropine. What guidelines will the nurse follow when administering atropine? A) Administer atropine 0.5 mg as an IV bolus every 3 to 5 minutes to a maximum of 3.0 mg. B) Administer atropine as a continuous infusion until symptoms resolve. C) Administer atropine as a continuous infusion to a maximum of 30 mg in 24 hours. D) Administer atropine 1.0 mg sublingually.

A Feedback: Atropine 0.5 mg given rapidly as an intravenous (IV) bolus every 3 to 5 minutes to a maximum total dose of 3.0 mg is the medication of choice in treating symptomatic sinus bradycardia. By this guideline, the other listed options are inappropriate.

A patient with a complex cardiac history is scheduled for transthoracic echocardiography. What should the nurse teach the patient in anticipation of this diagnostic procedure? A) The test is noninvasive, and nothing will be inserted into the patients body. B) The patients pain will be managed aggressively during the procedure. C) The test will provide a detailed profile of the hearts electrical activity. D) The patient will remain on bed rest for 1 to 2 hours after the test.

A Feedback: Before transthoracic echocardiography, the nurse informs the patient about the test, explaining that it is painless. The test does not evaluate electrophysiology and bed rest is unnecessary after the procedure.

The nurse is caring for a patient who has had a biventricular pacemaker implanted. When planning the patients care, the nurse should recognize what goal of this intervention? A) Resynchronization B) Defibrillation C) Angioplasty D) Ablation

A Feedback: Biventricular (both ventricles) pacing, also called resynchronization therapy, may be used to treat advanced heart failure that does not respond to medication. This type of pacing therapy is not called defibrillation, angioplasty, or ablation therapy.

A patient with primary hypertension comes to the clinic complaining of a gradual onset of blurry vision and decreased visual acuity over the past several weeks. The nurse is aware that these symptoms could be indicative of what? A) Retinal blood vessel damage B) Glaucoma C) Cranial nerve damage D) Hypertensive emergency

A Feedback: Blurred vision, spots in front of the eyes, and diminished visual acuity can mean retinal blood vessel damage indicative of damage elsewhere in the vascular system as a result of hypertension. Glaucoma and cranial nerve damage do not normally cause these symptoms. A hypertensive emergency would have a more rapid onset.

A patient with cardiovascular disease is being treated with amlodipine (Norvasc), a calcium channel blocking agent. The therapeutic effects of calcium channel blockers include which of the following? A) Reducing the hearts workload by decreasing heart rate and myocardial contraction B) Preventing platelet aggregation and subsequent thrombosis C) Reducing myocardial oxygen consumption by blocking adrenergic stimulation to the heart D) Increasing the efficiency of myocardial oxygen consumption, thus decreasing ischemia and relieving pain

A Feedback: Calcium channel blocking agents decrease sinoatrial node automaticity and atrioventricular node conduction, resulting in a slower heart rate and a decrease in the strength of the heart muscle contraction. These effects decrease the workload of the heart. Antiplatelet and anticoagulation medications are administered to prevent platelet aggregation and subsequent thrombosis, which impedes blood flow. Beta-blockers reduce myocardial consumption by blocking beta-adrenergic sympathetic stimulation to the heart. The result is reduced myocardial contractility (force of contraction) to balance the myocardium oxygen needs and supply. Nitrates reduce myocardial oxygen consumption, which decreases ischemia and relieves pain by dilating the veins and, in higher doses, the arteries.

When assessing venous disease in a patients lower extremities, the nurse knows that what test will most likely be ordered? A) Duplex ultrasonography B) Echocardiography C) Positron emission tomography (PET) D) Radiography

A Feedback: Duplex ultrasound may be used to determine the level and extent of venous disease as well as its chronicity. Radiographs (x-rays), PET scanning, and echocardiography are never used for this purpose as they do not allow visualization of blood flow.

The nurse and the other members of the team are caring for a patient who converted to ventricular fibrillation (VF). The patient was defibrillated unsuccessfully and the patient remains in VF. According to national standards, the nurse should anticipate the administration of what medication? A) Epinephrine 1 mg IV push B) Lidocaine 100 mg IV push C) Amiodarone 300 mg IV push D) Sodium bicarbonate 1 amp IV push

A Feedback: Epinephrine should be administered as soon as possible after the first unsuccessful defibrillation and then every 3 to 5 minutes. Antiarrhythmic medications such as amiodarone and licocaine are given if ventricular dysrhythmia persists.

The nurse is caring for a patient who is in the recovery room following the implantation of an ICD. The patient has developed ventricular tachycardia (VT). What should the nurse assess and document? A) ECG to compare time of onset of VT and onset of devices shock B) ECG so physician can see what type of dysrhythmia the patient has C) Patients level of consciousness (LOC) at the time of the dysrhythmia D) Patients activity at time of dysrhythmia

A Feedback: If the patient has an ICD implanted and develops VT or ventricular fibrillation, the ECG should be recorded to note the time between the onset of the dysrhythmia and the onset of the devices shock or antitachycardia pacing. This is a priority over LOC or activity at the time of onset.

A patient with advanced venous insufficiency is confined following orthopedic surgery. How can the nurse best prevent skin breakdown in the patients lower extremities? A) Ensure that the patients heels are protected and supported. B) Closely monitor the patients serum albumin and prealbumin levels. C) Perform gentle massage of the patients lower legs, as tolerated. D) Perform passive range-of-motion exercises once per shift.

A Feedback: If the patient is on bed rest, it is important to relieve pressure on the heels to prevent pressure ulcerations, since the heels are among the most vulnerable body regions. Monitoring blood work does not directly prevent skin breakdown, even though albumin is related to wound healing. Massage is not normally indicated and may exacerbate skin breakdown. Passive range- of-motion exercises do not directly reduce the risk of skin breakdown

The nurse is caring for a patient with severe left ventricular dysfunction who has been identified as being at risk for sudden cardiac death. What medical intervention can be performed that may extend the survival of the patient? A) Insertion of an implantable cardioverter defibrillator B) Insertion of an implantable pacemaker C) Administration of a calcium channel blocker D) Administration of a beta-blocker

A Feedback: In patients with severe left ventricular dysfunction and the possibility of life-threatening dysrhythmias, placement of an implantable cardioverter defibrillator (ICD) can prevent sudden cardiac death and extend survival. A pacemaker, a calcium channel blocker, and a beta-blocker are not medical interventions that may extend the survival of the patient with left ventricular dysfunction.

A nurse is creating an education plan for a patient with venous insufficiency. What measure should the nurse include in the plan? A) Avoiding tight-fitting socks. B) Limit activity whenever possible. C) Sleep with legs in a dependent position. D) Avoid the use of pressure stockings

A Feedback: Measures taken to prevent complications include avoiding tight-fitting socks and panty girdles; maintaining activities, such as walking, sleeping with legs elevated, and using pressure stockings. Not included in the teaching plan for venous insufficiency would be reducing activity, sleeping with legs dependent, and avoiding pressure stockings. Each of these actions exacerbates venous insufficiency.

A nurse is teaching an adult female patient about the risk factors for hypertension. What should the nurse explain as risk factors for primary hypertension? A) Obesity and high intake of sodium and saturated fat B) Diabetes and use of oral contraceptives C) Metabolic syndrome and smoking D) Renal disease and coarctation of the aorta

A Feedback: Obesity, stress, high intake of sodium or saturated fat, and family history are all risk factors for primary hypertension. Diabetes and oral contraceptives are risk factors for secondary hypertension. Metabolic syndrome, renal disease, and coarctation of the aorta are causes of secondary hypertension.

The nurse is admitting a 32-year-old woman to the presurgical unit. The nurse learns during the admission assessment that the patient takes oral contraceptives. Consequently, the nurses postoperative plan of care should include what intervention? A) Early ambulation and leg exercises B) Cessation of the oral contraceptives until 3 weeks postoperative C) Doppler ultrasound of peripheral circulation twice daily D) Dependent positioning of the patients extremities when at rest

A Feedback: Oral contraceptive use increases blood coagulability; with bed rest, the patient may be at increased risk of developing deep vein thrombosis. Leg exercises and early ambulation are among the interventions that address this risk. Assessment of peripheral circulation is important, but Doppler ultrasound may not be necessary to obtain these data. Dependent positioning increases the risk of venous thromboembolism (VTE). Contraceptives are not normally discontinued to address the risk of VTE in the short term.

The nurse is preparing a patient for cardiac surgery. During the procedure, the patients heart will be removed and a donor heart implanted at the vena cava and pulmonary veins. What procedure will this patient undergo? A) Orthotopic transplant B) Xenograft C) Heterotropic transplant D) Homograft

A Feedback: Orthotopic transplantation is the most common surgical procedure for cardiac transplantation. The recipients heart is removed, and the donor heart is implanted at the vena cava and pulmonary veins. Some surgeons still prefer to remove the recipients heart, leaving a portion of the recipients atria (with the vena cava and pulmonary veins) in place. Homografts, or allografts (i.e., human valves), are obtained from cadaver tissue donations and are used for aortic and pulmonic valve replacement. Xenografts and heterotropic transplantation are not terms used to describe heart transplantation.

A cardiovascular patient with a previous history of pulmonary embolism (PE) is experiencing a sudden onset of dyspnea, rapid breathing, and chest pain. The nurse recognizes the characteristic signs and symptoms of a PE. What is the nurses best action? A) Rapidly assess the patients cardiopulmonary status. B) Arrange for an ECG. C) Increase the height of the patients bed. D) Manage the patients anxiety.

A Feedback: Patient management in the event of a PE begins with cardiopulmonary assessment and intervention. This is a priority over ECG monitoring, management of anxiety, or repositioning of the patient, even though each of these actions may be appropriate and necessary.

A patient who has undergone valve replacement surgery is being prepared for discharge home. Because the patient will be discharged with a prescription for warfarin (Coumadin), the nurse should educate the patient about which of the following? A) The need for regularly scheduled testing of the patients International Normalized Ratio (INR) B) The need to learn to sleep in a semi-Fowlers position for the first 6 to 8 weeks to prevent emboli C) The need to avoid foods that contain vitamin K D) The need to take enteric-coated ASA on a daily basis

A Feedback: Patients who take warfarin (Coumadin) after valve replacement have individualized target INRs; usually between 2 and 3.5 for mitral valve replacement and 1.8 and 2.2 for aortic valve replacement. Natural sources of vitamin K do not normally need to be avoided and ASA is not indicated. Sleeping upright is unnecessary.

The nurse is providing patient education prior to a patients discharge home after treatment for HF. The nurse gives the patient a home care checklist as part of the discharge teaching. What should be included on this checklist? A) Know how to recognize and prevent orthostatic hypotension. B) Weigh yourself weekly at a consistent time of day. C) Measure everything you eat and drink until otherwise instructed. D) Limit physical activity to only those tasks that are absolutely necessary.

A Feedback: Patients with HF should be aware of the risks of orthostatic hypotension. Weight should be measured daily; detailed documentation of all forms of intake is not usually required. Activity should be gradually increased within the parameters of safety and comfort.

A patient admitted to the medical unit with HF is exhibiting signs and symptoms of pulmonary edema. The nurse is aware that positioning will promote circulation. How should the nurse best position the patient? A) In a high Fowlers position B) On the left side-lying position C) In a flat, supine position D) In the Trendelenburg position

A Feedback: Proper positioning can help reduce venous return to the heart. The patient is positioned upright. If the patient is unable to sit with the lower extremities dependent, the patient may be placed in an upright position in bed. The supine position and Trendelenburg positions will not reduce venous return, lower the output of the right ventricle, or decrease lung congestion. Similarly, side-lying does not promote circulation.

The nurse is calculating a cardiac patients pulse pressure. If the patients blood pressure is 122/76 mm Hg, what is the patients pulse pressure? A) 46 mm Hg B) 99 mm Hg C) 198 mm Hg D) 76 mm Hg

A Feedback: Pulse pressure is the difference between the systolic and diastolic pressure. In this case, this value is 46 mm Hg.

A patient is admitted to the critical care unit (CCU) with a diagnosis of cardiomyopathy. When reviewing the patients most recent laboratory results, the nurse should prioritize assessment of which of the following? A) Sodium B) AST, ALT, and bilirubin C) White blood cell differential D) BUN

A Feedback: Sodium is the major electrolyte involved with cardiomyopathy. Cardiomyopathy often leads to heart failure which develops, in part, from fluid overload. Fluid overload is often associated with elevated sodium levels. Consequently, sodium levels are followed more closely than other important laboratory values, including BUN, leukocytes, and liver function tests.

A patient has been diagnosed with a valvular disorder. The patient tells the nurse that he has read about numerous treatment options, including valvuloplasty. What should the nurse teach the patient about valvuloplasty? A) For some patients, valvuloplasty can be done in a cardiac catheterization laboratory. B) Valvuloplasty is a dangerous procedure, but it has excellent potential if it goes well. C) Valvuloplasty is open heart surgery, but this is very safe these days and normally requires only an overnight hospital stay. D) Its prudent to get a second opinion before deciding to have valvuloplasty.

A Feedback: Some valvuloplasty procedures do not require general anesthesia or cardiopulmonary bypass and can be performed in a cardiac catheterization laboratory or hybrid room. Open heart surgery is not required and the procedure does not carry exceptional risks that would designate it as being dangerous. Normally there is no need for the nurse to advocate for a second opinion.

A patient presents to the ED in distress and complaining of crushing chest pain. What is the nurses priority for assessment? A) Prompt initiation of an ECG B) Auscultation of the patients point of maximal impulse (PMI) C) Rapid assessment of the patients peripheral pulses D) Palpation of the patients cardiac apex

A Feedback: The 12-lead ECG provides information that assists in ruling out or diagnosing an acute MI. It should be obtained within 10 minutes from the time a patient reports pain or arrives in the ED. Each of the other listed assessments is valid, but ECG monitoring is the most time dependent priority.

A patient who is a candidate for an implantable cardioverter defibrillator (ICD) asks the nurse about the purpose of this device. What would be the nurses best response? A) To detect and treat dysrhythmias such as ventricular fibrillation and ventricular tachycardia B) To detect and treat bradycardia, which is an excessively slow heart rate C) To detect and treat atrial fibrillation, in which your heart beats too quickly and inefficiently D) To shock your heart if you have a heart attack at home

A Feedback: The ICD is a device that detects and terminates life-threatening episodes of ventricular tachycardia and ventricular fibrillation. It does not treat atrial fibrillation, MI, or bradycardia.

The nurse is performing a physical assessment on a patient suspected of having HF. The presence of what sound would signal the possibility of impending HF? A) An S3 heart sound B) Pleural friction rub C) Faint breath sounds D) A heart murmur

A Feedback: The heart is auscultated for an S3 heart sound, a sign that the heart is beginning to fail and that increased blood volume fills the ventricle with each beat. HF does not normally cause a pleural friction rub or murmurs. Changes in breath sounds occur, such as the emergence of crackles or wheezes, but faint breath sounds are less characteristic of HF.

The critical care nurse is caring for a patient who has been experiencing bradycardia after cardiovascular surgery. The nurse knows that the heart rate is determined by myocardial cells with the fastest inherent firing rate. Under normal circumstances where are these cells located? A) SA node B) AV node C) Bundle of His D) Purkinje cells

A Feedback: The heart rate is determined by the myocardial cells with the fastest inherent firing rate. Under normal circumstances, the SA node has the highest inherent rate (60 to 100 impulses per minute).

The clinic nurse is caring for a 57-year-old client who reports experiencing leg pain whenever she walks several blocks. The patient has type 1 diabetes and has smoked a pack of cigarettes every day for the past 40 years. The physician diagnoses intermittent claudication. The nurse should provide what instruction about long-term care to the client? A) Be sure to practice meticulous foot care. B) Consider cutting down on your smoking. C) Reduce your activity level to accommodate your limitations. D) Try to make sure you eat enough protein.

A Feedback: The patient with peripheral vascular disease or diabetes should receive education or reinforcement about skin and foot care. Intermittent claudication and other chronic peripheral vascular diseases reduce oxygenation to the feet, making them susceptible to injury and poor healing; therefore, meticulous foot care is essential. The patient should stop smokingnot just cut downbecause nicotine is a vasoconstrictor. Daily walking benefits the patient with intermittent claudication. Increased protein intake will not alleviate the patients symptoms.

A patients declining cardiac status has been attributed to decreased cardiac action potential. Interventions will be aimed at restoring what aspect of cardiac physiology? A) The cycle of depolarization and repolarization B) The time it takes from the firing of the SA node to the contraction of the ventricles C) The time between the contraction of the atria and the contraction of the ventricles D) The cycle of the firing of the AV node and the contraction of the myocardium

A Feedback: This exchange of ions creates a positively charged intracellular space and a negatively charged extracellular space that characterizes the period known as depolarization. Once depolarization is complete, the exchange of ions reverts to its resting state; this period is known as repolarization. The repeated cycle of depolarization and repolarization is called the cardiac action potential.

A patient has been living with dilated cardiomyopathy for several years but has experienced worsening symptoms despite aggressive medical management. The nurse should anticipate what potential treatment? A) Heart transplantation B) Balloon valvuloplasty C) Cardiac catheterization D) Stent placement

A Feedback: When heart failure progresses and medical treatment is no longer effective, surgical intervention, including heart transplantation, is considered. Valvuloplasty, stent placement, and cardiac catheterization will not address the pathophysiology of cardiomyopathy.

The triage nurse in the ED assesses a 66-year-old male patient who presents to the ED with complaints of midsternal chest pain that has lasted for the last 5 hours. If the patients symptoms are due to an MI, what will have happened to the myocardium? A) It may have developed an increased area of infarction during the time without treatment. B) It will probably not have more damage than if he came in immediately. C) It may be responsive to restoration of the area of dead cells with proper treatment. D) It has been irreparably damaged, so immediate treatment is no longer necessary.

A Feedback: When the patient experiences lack of oxygen to myocardium cells during an MI, the sooner treatment is initiated, the more likely the treatment will prevent or minimize myocardial tissue necrosis. Delays in treatment equate with increased myocardial damage. Despite the length of time the symptoms have been present, treatment needs to be initiated immediately to minimize further damage. Dead cells cannot be restored by any means.

A cardiac patients resistance to left ventricular filling has caused blood to back up into the patients circulatory system. What health problem is likely to result? A) Acute pulmonary edema B) Right-sided HF C) Right ventricular hypertrophy D) Left-sided HF

A Feedback: With increased resistance to left ventricular filling, blood backs up into the pulmonary circulation. The patient quickly develops pulmonary edema from the blood volume overload in the lungs. When the blood backs up into the pulmonary circulation, right-sided HF, left-sided HF, and right ventricular hypertrophy do not directly occur.

The nurse is caring for a patient with a large venous leg ulcer. What intervention should the nurse implement to promote healing and prevent infection? A) Provide a high-calorie, high-protein diet. B) Apply a clean occlusive dressing once daily and whenever soiled. C) Irrigate the wound with hydrogen peroxide once daily. D) Apply an antibiotic ointment on the surrounding skin with each dressing change.

A Feedback: Wound healing is highly dependent on adequate nutrition. The diet should be sufficiently high in calories and protein. Antibiotic ointments are not normally used on the skin surrounding a leg ulcer and occlusive dressings can exacerbate impaired blood flow. Hydrogen peroxide is not normally used because it can damage granulation tissue.

A community health nurse is presenting an educational event and is addressing several health problems, including rheumatic heart disease. What should the nurse describe as the most effective way to prevent rheumatic heart disease? A) Recognizing and promptly treating streptococcal infections B) Prophylactic use of calcium channel blockers in high-risk populations C) Adhering closely to the recommended child immunization schedule D) Smoking cessation

A Group A streptococcus can cause rheumatic heart fever, resulting in rheumatic endocarditis. Being aware of signs and symptoms of streptococcal infections, identifying them quickly, and treating them promptly, are the best preventative techniques for rheumatic endocarditis. Smoking cessation, immunizations, and calcium channel blockers will not prevent rheumatic heart disease.

A patient who is at high risk for developing intracardiac thrombi has been placed on long-term anticoagulation. What aspect of the patients health history creates a heightened risk of intracardiac thrombi? A) Atrial fibrillation B) Infective endocarditis C) Recurrent pneumonia D) Recent surgery

A Intracardiac thrombi are especially common in patients with atrial fibrillation, because the atria do not contract forcefully and blood flows slowly and turbulently, increasing the likelihood of thrombus formation. Endocarditis, pneumonia, and recent surgery do not normally cause an increased risk for intracardiac thrombi formation.

A patient the nurse is caring for has a permanent pacemaker implanted with the identification code beginning with VVI. What does this indicate? A) Ventricular paced, ventricular sensed, inhibited B) Variable paced, ventricular sensed, inhibited C) Ventricular sensed, ventricular situated, implanted D) Variable sensed, variable paced, inhibited

A The identification of VVI indicates ventricular paced, ventricular sensed, inhibited.

The student nurse is preparing a teaching plan for a patient being discharged status post MI. What should the student include in the teaching plan? (Mark all that apply.) A) Need for careful monitoring for cardiac symptoms B) Need for carefully regulated exercise C) Need for dietary modifications D) Need for early resumption of prediagnosis activity E) Need for increased fluid intake

A, B, C Feedback: Dietary modifications, exercise, weight loss, and careful monitoring are important strategies for managing three major cardiovascular risk factors: hyperlipidemia, hypertension, and diabetes. There is no need to increase fluid intake and activity should be slowly and deliberately increased.

The patient has just returned to the floor after balloon valvuloplasty of the aortic valve and the nurse is planning appropriate assessments. The nurse should know that complications following this procedure include what? Select all that apply. A) Emboli B) Mitral valve damage C) Ventricular dysrhythmia D) Atrial-septal defect E) Plaque formation

A, B, C Feedback: Possible complications include aortic regurgitation, emboli, ventricular perforation, rupture of the aortic valve annulus, ventricular dysrhythmia, mitral valve damage, and bleeding from the catheter insertion sites. Atrial-septal defect and plaque formation are not complications of a balloon valvuloplasty.

The nurse is assessing a patient with acute coronary syndrome (ACS). The nurse includes a careful history in the assessment, especially with regard to signs and symptoms. What signs and symptoms are suggestive of ACS? Select all that apply. A) Dyspnea B) Unusual fatigue C) Hypotension D) Syncope E) Peripheral cyanosis

A, B, D Feedback: Systematic assessment includes a careful history, particularly as it relates to symptoms: chest pain or discomfort, difficulty breathing (dyspnea), palpitations, unusual fatigue, faintness (syncope), or sweating (diaphoresis). Each symptom must be evaluated with regard to time, duration, and the factors that precipitate the symptom and relieve it, and in comparison with previous symptoms. Hypotension and peripheral cyanosis are not typically associated with ACS.

When hemodynamic monitoring is ordered for a patient, a catheter is inserted into the appropriate blood vessel or heart chamber. When assessing a patient who has such a device in place, the nurse should check which of the following components? Select all that apply. A) A transducer B) A flush system C) A leveler D) A pressure bag E) An oscillator

A, B, D Feedback: To perform hemodynamic monitoring, a CVP, pulmonary artery, or arterial catheter is introduced into the appropriate blood vessel or heart chamber. It is connected to a pressure monitoring system that has several components. Included among these are a transducer, a flush system, and a pressure bag. A pressure monitoring system does not have a leveler or an oscillator.

A critical care nurse is caring for a patient with a hemodynamic monitoring system in place. For what complications should the nurse assess? Select all that apply. A) Pneumothorax B) Infection C) Atelectasis D) Bronchospasm E) Air embolism

A, B, E Complications from use of hemodynamic monitoring systems are uncommon, but can include pneumothorax, infection, and air embolism. Complications of hemodynamic monitoring systems do not include atelectasis or bronchospasm.

The nurse is evaluating a patients diagnosis of arterial insufficiency with reference to the adequacy of the patients blood flow. On what physiological variables does adequate blood flow depend? Select all that apply. A) Efficiency of heart as a pump B) Adequacy of circulating blood volume C) Ratio of platelets to red blood cells D) Size of red blood cells E) Patency and responsiveness of the blood vessels

A, B, E Feedback: Adequate blood flow depends on the efficiency of the heart as a pump, the patency and responsiveness of the blood vessels, and the adequacy of circulating blood volume. Adequacy of blood flow does not primarily depend on the size of red cells or their ratio to the number of platelets.

The patient has a homocysteine level ordered. What aspects of this test should inform the nurses care? Select all that apply. A) A 12-hour fast is necessary before drawing the blood sample. B) Recent inactivity can depress homocysteine levels. C) Genetic factors can elevate homocysteine levels. Test Bank - Brunner & Suddarth's Textbook of Medical-Surgical Nursing 14e (Hinkle 2017) 504 D) A diet low in folic acid elevates homocysteine levels. E) An ECG should be performed immediately before drawing a sample.

A, C, D Feedback: Genetic factors and a diet low in folic acid, vitamin B6, and vitamin B12 are associated with elevated homocysteine levels. A 12-hour fast is necessary before drawing a blood sample for an accurate serum measurement. An ECG is unnecessary and recent inactivity does not influence the results of the test.

The nurse is creating a care plan for a patient diagnosed with HF. When addressing the problem of anxiety, what interventions should the nurse include in the care plan? Select all that apply. A) Facilitate the presence of friends and family whenever possible. B) Teach the patient about the harmful effects of anxiety on cardiac function. C) Provide supplemental oxygen, as needed. D) Provide validation of the patients expressions of anxiety. E) Administer benzodiazepines two to three times daily.

A, C, D Feedback: The nurse should empathically validate the patients sensations of anxiety. The presence of friends and family are frequently beneficial and oxygen supplementation promotes comfort. Antianxiety medications may be necessary for some patients, but alternative methods of relief should be prioritized. As well, medications are administered on a PRN basis. Teaching the patient about the potential harms of anxiety is likely to exacerbate, not relieve, the problem.

The nurse providing care for a patient post PTCA knows to monitor the patient closely. For what complications should the nurse monitor the patient? Select all that apply. A) Abrupt closure of the coronary artery B) Venous insufficiency C) Bleeding at the insertion site D) Retroperitoneal bleeding E) Arterial occlusion

A, C, D, E Feedback: Complications after the procedure may include abrupt closure of the coronary artery and vascular complications, such as bleeding at the insertion site, retroperitoneal bleeding, hematoma, and arterial occlusion, as well as acute renal failure. Venous insufficiency is not a postprocedure complication of a PTCA.

The cardiac nurse is caring for a patient who has been diagnosed with dilated cardiomyopathy (DCM). Echocardiography is likely to reveal what pathophysiological finding? A) Decreased ejection fraction B) Decreased heart rate C) Ventricular hypertrophy D) Mitral valve regurgitation

Ans: A Feedback: DCM is distinguished by significant dilation of the ventricles without simultaneous hypertrophy. The ventricles have elevated systolic and diastolic volumes, but a decreased ejection fraction. Bradycardia and mitral valve regurgitation do not typically occur in patients with DCM.

The nurse on the hospitals infection control committee is looking into two cases of hospital-acquired infective endocarditis among a specific classification of patients. What classification of patients would be at greatest risk for hospital-acquired endocarditis? A) Hemodialysis patients B) Patients on immunoglobulins C) Patients who undergo intermittent urinary catheterization D) Children under the age of 12

Ans: A Feedback: Hospital-acquired infective endocarditis occurs most often in patients with debilitating disease or indwelling catheters and in patients who are receiving hemodialysis or prolonged IV fluid or antibiotic therapy. Patients taking immunosuppressive medications or corticosteroids are more susceptible to fungal endocarditis. Patients on immunoglobulins, those who need in and out catheterization, and children are not at increased risk for nosocomial infective endocarditis.

The nurse is caring for a patient who returned from the tropics a few weeks ago and who sought care with signs and symptoms of lymphedema. The nurses plan of care should prioritize what nursing diagnosis? A) Risk for infection related to lymphedema B) Disturbed body image related to lymphedema C) Ineffective health maintenance related to lymphedema D) Risk for deficient fluid volume related to lymphedema

Ans: A Feedback: Lymphedema, which is caused by accumulation of lymph in the tissues, constitutes a significant risk for infection. The patients body image is likely to be disturbed, and the nurse should address this, but infection is a more significant threat to the patients physiological well-being. Lymphedema is unrelated to ineffective health maintenance and deficient fluid volume is not a significant risk.

The nurse is reviewing the medication administration record of a patient diagnosed with systolic HF. What medication should the nurse anticipate administering to this patient? A) A beta-adrenergic blocker B) An antiplatelet aggregator C) A calcium channel blocker D) A nonsteroidal anti-inflammatory drug (NSAID)

Ans: A Feedback: Several medications are routinely prescribed for systolic HF, including ACE inhibitors, beta-blockers, diuretics, and digitalis. Calcium channel blockers, antiplatelet aggregators, and NSAIDs are not commonly prescribed.

The nurse is caring for an adult patient who has gone into ventricular fibrillation. When assisting with defibrillating the patient, what must the nurse do? A) Maintain firm contact between paddles and patient skin. B) Apply a layer of water as a conducting agent. C) Call all clear once before discharging the defibrillator. D) Ensure the defibrillator is in the sync mode.

Ans: A Feedback: When defibrillating an adult patient, the nurse should maintain good contact between the paddles and the patients skin to prevent arcing, apply an appropriate conducting agent (not water) between the skin and the paddles, and ensure the defibrillator is in the nonsync mode. Clear should be called three times before discharging the paddles.

The nurse is teaching a patient about some of the health consequences of uncontrolled hypertension. What health problems should the nurse describe? Select all that apply. A) Transient ischemic attacks B) Cerebrovascular accident C) Retinal hemorrhage D) Venous insufficiency E) Right ventricular hypertrophy

Ans: A, B, C Feedback: Potential complications of hypertension include the following: left ventricular hypertrophy; MI; heart failure; transient ischemic attacks (TIAs); cerebrovascular accident; renal insufficiency and failure; and retinal hemorrhage. Venous insufficiency and right ventricular hypertrophy are not potential complications of uncontrolled hypertension.

Most individuals who have mitral valve prolapse never have any symptoms, although this is not the case for every patient. What symptoms might a patient have with mitral valve prolapse? Select all that apply. A) Anxiety B) Fatigue C) Shoulder pain D) Tachypnea E) Palpitations

Ans: A, B, E Feedback: Most people who have mitral valve prolapse never have symptoms. A few have symptoms of fatigue, shortness of breath, lightheadedness, dizziness, syncope, palpitations, chest pain, and anxiety. Hyperpnea and shoulder pain are not characteristic symptoms of mitral valve prolapse.

The nurse is planning the care of a patient with HF. The nurse should identify what overall goals of this patients care? (Select all that apply) A) Improve functional status B) Prevent endocarditis. C) Extend survival. D) Limit physical activity. E) Relieve patient symptoms.

Ans: A, C, E Feedback: The overall goals of management of HF are to relieve the patients symptoms, to improve functional status and quality of life, and to extend survival. Activity limitations should be accommodated, but reducing activity is no

The nurse is reviewing the echocardiography results of a patient who has just been diagnosed with dilated cardiomyopathy (DCM). What changes in heart structure characterize DCM? A) Dilated ventricles with atrophy of the ventricles B) Dilated ventricles without hypertrophy of the ventricles C) Dilation and hypertrophy of all four heart chambers D) Dilation of the atria and hypertrophy of the ventricles

Ans: B Feedback: DCM is characterized by significant dilation of the ventricles without significant concomitant hypertrophy and systolic dysfunction. The ventricles do not atrophy in patients with DCM.

The nurses comprehensive assessment of a patient who has HF includes evaluation of the patients hepatojugular reflux. What action should the nurse perform during this assessment? A) Elevate the patients head to 90 degrees. B) Press the right upper abdomen. C) Press above the patients symphysis pubis. D) Lay the patient flat in bed.

Ans: B Feedback: Hepatojugular reflux, a sign of right-sided heart failure, is assessed with the head of the bed at a 45- degree angle. As the right upper abdomen (the area over the liver) is compressed for 30 to 40 seconds, the nurse observes the internal jugular vein. If the internal jugular vein becomes distended, a patient has positive hepatojugular reflux.

The nurse is assessing a patient who is known to have right-sided HF. What assessment finding is most consistent with this patients diagnosis? A) Pulmonary edema B) Distended neck veins C) Dry cough D) Orthopnea

Ans: B Feedback: Right-sided HF may manifest by distended neck veins, dependent edema, hepatomegaly, weight gain, ascites, anorexia, nausea, nocturia, and weakness. The other answers do not apply.

A patient newly admitted to the telemetry unit is experiencing progressive fatigue, hemoptysis, and dyspnea. Diagnostic testing has revealed that these signs and symptoms are attributable to pulmonary venous hypertension. What valvular disorder should the nurse anticipate being diagnosed in this patient? A) Aortic regurgitation B) Mitral stenosis C) Mitral valve prolapse D) Aortic stenosis

Ans: B Feedback: The first symptom of mitral stenosis is often dyspnea on exertion as a result of pulmonary venous hypertension. Symptoms usually develop after the valve opening is reduced by one-third to one-half its usual size. Patients are likely to show progressive fatigue as a result of low cardiac output. The enlarged left atrium may create pressure on the left bronchial tree, resulting in a dry cough or wheezing. Patients may expectorate blood (i.e., hemoptysis) or experience palpitations, orthopnea, paroxysmal nocturnal dyspnea (PND), and repeated respiratory infections. Pulmonary venous hypertension is not typically caused by aortic regurgitation, mitral valve prolapse, or aortic stenosis.

The nurse is planning the care of a patient who has been diagnosed with hypertension, but who otherwise enjoys good health. When assessing the response to an antihypertensive drug regimen, what blood pressure would be the goal of treatment? A) 156/96 mm Hg or lower B) 140/90 mm Hg or lower C) Average of 2 BP readings of 150/80 mm Hg D) 120/80 mm Hg or lower

Ans: B Feedback: The goal of antihypertensive drug therapy is a BP of 140/90 mm Hg or lower. A pressure of 130/80 mm Hg is the goal for patients with diabetes or chronic kidney disease.

A patient has been admitted to the medical unit with signs and symptoms suggestive of endocarditis. The physicians choice of antibiotics would be primarily based on what diagnostic test? A) Echocardiography B) Blood cultures C) Cardiac aspiration D) Complete blood count

Ans: B Feedback: To help determine the causative organisms and the most effective antibiotic treatment for the patient, blood cultures are taken. A CBC can help establish the degree and stage of infection, but not the causative microorganism. Echocardiography cannot indicate the microorganisms causing the infection. Cardiac aspiration is not a diagnostic test.

The prevention of VTE is an important part of the nursing care of high-risk patients. When providing patient teaching for these high-risk patients, the nurse should advise lifestyle changes, including which of the following? Select all that apply. A) High-protein diet B) Weight loss C) Regular exercise D) Smoking cessation E) Calcium and vitamin D supplementation

Ans: B, C, D Feedback: Patients at risk for VTE should be advised to make lifestyle changes, as appropriate, which may include weight loss, smoking cessation, and regular exercise. Increased protein intake and supplementation with vitamin D and calcium do not address the main risk factors for VTE.

The critical care nurse is caring for a patient who is in cardiogenic shock. What assessments must the nurse perform on this patient? Select all that apply. A) Platelet level B) Fluid status C) Cardiac rhythm D) Action of medications E) Sputum volume

Ans: B, C, D Feedback: The critical care nurse must carefully assess the patient in cardiogenic shock, observe the cardiac rhythm, monitor hemodynamic parameters, monitor fluid status, and adjust medications and therapies based on the assessment data. Platelet levels and sputum production are not major assessment parameters in a patient who is experiencing cardiogenic shock.

The nurse is caring for a patient with systolic HF whose previous adverse reactions preclude the safe use of ACE inhibitors. The nurse should anticipate that the prescriber may choose what combination of drugs? A) Loop diuretic and antiplatelet aggregator B) Loop diuretic and calcium channel blocker C) Combination of hydralazine and isosorbide dinitrate D) Combination of digoxin and normal saline

Ans: C Feedback: A combination of hydralazine and isosorbide dinitrate may be an alternative for patients who cannot take ACE inhibitors. Antiplatelet aggregators, calcium channel blockers, and normal saline are not typically prescribed.

The nurse is caring for an 84-year-old man who has just returned from the OR after inguinal hernia repair. The OR report indicates that the patient received large volumes of IV fluids during surgery and the nurse recognizes that the patient is at risk for left-sided heart failure. What signs and symptoms would indicate left-sided heart failure? A) Jugular vein distention B) Right upper quadrant pain C) Bibasilar fine crackles D) Dependent edema

Ans: C Feedback: Bibasilar fine crackles are a sign of alveolar fluid, a sequela of left ventricular fluid, or pressure overload. Jugular vein distention, right upper quadrant pain (hepatomegaly), and dependent edema are caused by right-sided heart failure, usually a chronic condition.

A 17-year-old boy is being treated in the ICU after going into cardiac arrest during a football practice. Diagnostic testing reveals cardiomyopathy as the cause of the arrest. What type of cardiomyopathy is particularly common among young people who appear otherwise healthy? A) Dilated cardiomyopathy (DCM). B) Arrhythmogenic right ventricular cardiomyopathy (ARVC) C) Hypertrophic cardiomyopathy (HCM) D) Restrictive or constrictive cardiomyopathy (RCM)

Ans: C Feedback: With HCM, cardiac arrest (i.e., sudden cardiac death) may be the initial manifestation in young people, including athletes. DCM, ARVC, and RCM are not typically present in younger adults who appear otherwise healthy.

A patient comes to the walk-in clinic with complaints of pain in his foot following stepping on a roofing nail 4 days ago. The patient has a visible red streak running up his foot and ankle. What health problem should the nurse suspect? A) Cellulitis B) Local inflammation C) Elephantiasis D) Lymphangitis

Ans: D Feedback: Lymphangitis is an acute inflammation of the lymphatic channels. It arises most commonly from a focus of infection in an extremity. Usually, the infectious organism is hemolytic streptococcus. The characteristic red streaks that extend up the arm or the leg from an infected wound outline the course of the lymphatic vessels as they drain. Cellulitis is caused by bacteria, which cause a generalized edema in the subcutaneous tissues surrounding the affected area. Local inflammation would not present with red streaks in the lymphatic channels. Elephantiasis is transmitted by mosquitoes that carry parasitic worm larvae; the parasites obstruct the lymphatic channels and results in gross enlargement of the limbs.

The nursing lab instructor is teaching student nurses how to take blood pressure. To ensure accurate measurement, the lab instructor would teach the students to avoid which of the following actions? A) Measuring the BP after the patient has been seated quietly for more than 5 minutes B) Taking the BP at least 10 minutes after nicotine or coffee ingestion C) Using a cuff with a bladder that encircles at least 80% of the limb D) Using a bare forearm supported at heart level on a firm surface

B Blood pressures should be taken with the patient seated with arm bare, supported, and at heart level. The patient should not have smoked tobacco or taken caffeine in the 30 minutes preceding the measurement. The patient should rest quietly for 5 minutes before the reading is taken. The cuff bladder should encircle at least 80% of the limb being measured and have a width of at least 40% of limb circumference. Using a cuff that is too large results in a lower BP and a cuff that is too small will give a higher BP measurement.

A patient with an occluded coronary artery is admitted and has an emergency percutaneous transluminal coronary angioplasty (PTCA). The patient is admitted to the cardiac critical care unit after the PTCA. For what complication should the nurse most closely monitor the patient? A) Hyperlipidemia B) Bleeding at insertion site C) Left ventricular hypertrophy D) Congestive heart failure

B Complications of PTCA may include bleeding at the insertion site, abrupt closure of the artery, arterial thrombosis, and perforation of the artery. Complications do not include hyperlipidemia, left ventricular hypertrophy, or congestive heart failure; each of these problems takes an extended time to develop and none is emergent.

The nurse is auscultating the breath sounds of a patient with pericarditis. What finding is most consistent with this diagnosis? A) Wheezes B) Friction rub C) Fine crackles D) Coarse crackles

B Feedback: A pericardial friction rub is diagnostic of pericarditis. Crackles are associated with pulmonary edema and fluid accumulation, whereas wheezes signal airway constriction; neither of these occurs with pericarditis.

An occupational health nurse is providing an educational event and has been asked by an administrative worker about the risk of varicose veins. What should the nurse suggest as a proactive preventative measure for varicose veins? A) Sit with crossed legs for a few minutes each hour to promote relaxation. B) Walk for several minutes every hour to promote circulation. C) Elevate the legs when tired. D) Wear snug-fitting ankle socks to decrease edema.

B Feedback: A proactive approach to preventing varicose veins would be to walk for several minutes every hour to promote circulation. Sitting with crossed legs may promote relaxation, but it is contraindicated for patients with, or at risk for, varicose veins. Elevating the legs only helps blood passively return to the heart and does not help maintain the competency of the valves in the veins. Wearing tight ankle socks is contraindicated for patients with, or at risk for, varicose veins; socks that are below the muscles of the calf do not promote venous return, the socks simply capture the blood and promote venous stasis.

While assessing a patient the nurse notes that the patients ankle-brachial index (ABI) of the right leg is 0.40. How should the nurse best respond to this assessment finding? A) Assess the patients use of over-the-counter dietary supplements. B) Implement interventions relevant to arterial narrowing. C) Encourage the patient to increase intake of foods high in vitamin K. D) Adjust the patients activity level to accommodate decreased coronary output.

B Feedback: ABI is used to assess the degree of stenosis of peripheral arteries. An ABI of less than 1.0 indicates possible claudication of the peripheral arteries. It does not indicate inadequate coronary output. There is no direct indication for changes in vitamin K intake and OTC medications are not likely causative.

A patient has been scheduled for cardiovascular computed tomography (CT) with contrast. To prepare the patient for this test, what action should the nurse perform? A) Keep the patient NPO for at least 6 hours prior to the test. B) Establish peripheral IV access. C) Limit the patients activity for 2 hours before the test. D) Teach the patient to perform incentive spirometry.

B Feedback: An IV is necessary if contrast is to be used to enhance the images of the CT. The patient does not need to fast or limit his or her activity. Incentive spirometry is not relevant to this diagnostic test.

The staff educator is presenting a workshop on valvular disorders. When discussing the pathophysiology of aortic regurgitation the educator points out the need to emphasize that aortic regurgitation causes what? A) Cardiac tamponade B) Left ventricular hypertrophy C) Right-sided heart failure D) Ventricular insufficiency

B Feedback: Aortic regurgitation eventually causes left ventricular hypertrophy. In aortic regurgitation, blood from the aorta returns to the left ventricle during diastole in addition to the blood normally delivered by the left atrium. The left ventricle dilates, trying to accommodate the increased volume of blood. Aortic regurgitation does not cause cardiac tamponade, right-sided heart failure, or ventricular insufficiency.

A patient presents to the ED complaining of increasing shortness of breath. The nurse assessing the patient notes a history of left-sided HF. The patient is agitated and occasionally coughing up pink-tinged, foamy sputum. The nurse should recognize the signs and symptoms of what health problem? A) Right-sided heart failure B) Acute pulmonary edema C) Pneumonia D) Cardiogenic shock

B Feedback: Because of decreased contractility and increased fluid volume and pressure in patients with HF, fluid may be driven from the pulmonary capillary beds into the alveoli, causing pulmonary edema and signs and symptoms described. In right-sided heart failure, the patient exhibits hepatomegaly, jugular vein distention, and peripheral edema. In pneumonia, the patient would have a temperature spike, and sputum that varies in color. Cardiogenic shock would show signs of hypotension and tachycardia.

A nurse has taken on the care of a patient who had a coronary artery stent placed yesterday. When reviewing the patients daily medication administration record, the nurse should anticipate administering what drug? A) Ibuprofen B) Clopidogrel C) Dipyridamole D) Acetaminophen

B Feedback: Because of the risk of thrombus formation within the stent, the patient receives antiplatelet medications, usually aspirin and clopidogrel. Ibuprofen and acetaminophen are not antiplatelet drugs. Dipyridamole is not the drug of choice following stent placement.

A patient has returned to the cardiac care unit after having a permanent pacemaker implantation. For which potential complication should the nurse most closely assess this patient? A) Chest pain B) Bleeding at the implantation site C) Malignant hyperthermia D) Bradycardia

B Feedback: Bleeding, hematomas, local infections, perforation of the myocardium, and tachycardia are complications of pacemaker implantations. The nurse should monitor for chest pain and bradycardia, but bleeding is a more common immediate complication. Malignant hyperthermia is unlikely because it is a response to anesthesia administration.

The nurse is caring for a patient who has a history of heart disease. What factor should the nurse identify as possibly contributing to a decrease in cardiac output? A) A change in position from standing to sitting B) A heart rate of 54 bpm C) A pulse oximetry reading of 94% D) An increase in preload related to ambulation

B Feedback: Cardiac output is computed by multiplying the stroke volume by the heart rate. Cardiac output can be affected by changes in either stroke volume or heart rate, such as a rate of 54 bpm. An increase in preload will lead to an increase in stroke volume. A pulse oximetry reading of 94% does not indicate hypoxemia, as hypoxia can decrease contractility. Transitioning from standing to sitting would more likely increase rather than decrease cardiac output.

The nurse overseeing care in the ICU reviews the shift report on four patients. The nurse recognizes which patient to be at greatest risk for the development of cardiogenic shock? A) The patient admitted with acute renal failure B) The patient admitted following an MI C) The patient admitted with malignant hypertension D) The patient admitted following a stroke

B Feedback: Cardiogenic shock may occur following an MI when a large area of the myocardium becomes ischemic, necrotic, and hypokinetic. It also can occur as a result of end-stage heart failure, cardiac tamponade, pulmonary embolism, cardiomyopathy, and dysrhythmias. While patients with acute renal failure are at risk for dysrhythmias and patients experiencing a stroke are at risk for thrombus formation, the patient admitted following an MI is at the greatest risk for development of cardiogenic shock when compared with the other listed diagnoses.

A patient is scheduled for catheter ablation therapy. When describing this procedure to the patients family, the nurse should address what aspect of the treatment? A) Resetting of the hearts contractility B) Destruction of specific cardiac cells C) Correction of structural cardiac abnormalities D) Clearance of partially occluded coronary arteries

B Feedback: Catheter ablation destroys specific cells that are the cause or central conduction route of a tachydysrhythmia. It does not reset the hearts contractility and it does not address structural or vascular abnormalities.

A resident of a long-term care facility has complained to the nurse of chest pain. What aspect of the residents pain would be most suggestive of angina as the cause? A) The pain is worse when the resident inhales deeply. B) The pain occurs immediately following physical exertion. C) The pain is worse when the resident coughs. D) The pain is most severe when the resident moves his upper body.

B Feedback: Chest pain associated with angina is often precipitated by physical exertion. The other listed aspects of chest pain are more closely associated with noncardiac etiologies.

Which assessment would be most appropriate for a patient who is receiving a loop diuretic for HF? A) Monitor liver function studies B) Monitor for hypotension C) Assess the patients vitamin D intake D) Assess the patient for hyperkalemia

B Feedback: Diuretic therapy increases urine output and decreases blood volume, which places the patient at risk of hypotension. Patients are at risk of losing potassium with loop diuretic therapy and need to continue with potassium in their diet; hypokalemia is a consequent risk. Liver function is rarely compromised by diuretic therapy and vitamin D intake is not relevant.

The nurse is caring for an older adult with a diagnosis of hypertension who is being treated with a diuretic and beta-blocker. Which of the following should the nurse integrate into the management of this clients hypertension? A) Ensure that the patient receives a larger initial dose of antihypertensive medication due to impaired absorption. B) Pay close attention to hydration status because of increased sensitivity to extracellular volume depletion. C) Recognize that an older adult is less likely to adhere to his or her medication regimen than a younger patient. D) Carefully assess for weight loss because of impaired kidney function resulting from normal aging.

B Feedback: Elderly people have impaired cardiovascular reflexes and thus are more sensitive to extracellular volume depletion caused by diuretics. The nurse needs to assess hydration status, low BP, and postural hypotension carefully. Older adults may have impaired absorption, but they do not need a higher initial dose of an antihypertensive than a younger person. Adherence to treatment is not necessarily linked to age. Kidney function and absorption decline with age; less, rather than more antihypertensive medication is prescribed. Weight gain is not necessarily indicative of kidney function decline.

The nurse is addressing exercise and physical activity during discharge education with a patient diagnosed with HF. What should the nurse teach this patient about exercise? A) Do not exercise unsupervised. B) Eventually aim to work up to 30 minutes of exercise each day. C) Slow down if you get dizzy or short of breath. D) Start your exercise program with high-impact activities.

B Feedback: Eventually, a total of 30 minutes of physical activity every day should be encouraged. Supervision is not necessarily required and the emergence of symptoms should prompt the patient to stop exercising, not simply to slow the pace. Low-impact activities should be prioritized.

The nurse is caring for an acutely ill patient who has central venous pressure monitoring in place. What intervention should be included in the care plan of a patient with CVP in place? A) Apply antibiotic ointment to the insertion site twice daily. B) Change the site dressing whenever it becomes visibly soiled. C) Perform passive range-of-motion exercises to prevent venous stasis. D) Aspirate blood from the device once daily to test pH.

B Feedback: Gauze dressings should be changed every 2 days or transparent dressings at least every 7 days and whenever dressings become damp, loosened, or visibly soiled. Passive ROM exercise is not indicated and it is unnecessary and inappropriate to aspirate blood to test it for pH. Antibiotic ointments are contraindicated.

The physician has ordered a high-sensitivity C-reactive protein (hs-CRP) drawn on a patient. The results of this test will allow the nurse to evaluate the role of what process that is implicated in the development of atherosclerosis? A) Immunosuppression B) Inflammation C) Infection D) Hemostasis

B Feedback: High-sensitivity CRP is a protein produced by the liver in response to systemic inflammation. Inflammation is thought to play a role in the development and progression of atherosclerosis.

The ED nurse is caring for a patient who has gone into cardiac arrest. During external defibrillation, what action should the nurse perform? A) Place gel pads over the apex and posterior chest for better conduction. B) Ensure no one is touching the patient at the time shock is delivered. C) Continue to ventilate the patient via endotracheal tube during the procedure. D) Allow at least 3 minutes between shocks.

B Feedback: In external defibrillation, both paddles may be placed on the front of the chest, which is the standard paddle placement. Whether using pads, or paddles, the nurse must observe two safety measures. First, maintain good contact between the pads or paddles and the patients skin to prevent leaking. Second, ensure that no one is in contact with the patient or with anything that is touching the patient when the defibrillator is discharged, to minimize the chance that electrical current will be conducted to anyone other than the patient. Ventilation should be stopped during defibrillation.

The nurse is caring for a patient with mitral stenosis who is scheduled for a balloon valvuloplasty. The patient tells the nurse that he is unsure why the surgeon did not opt to replace his damaged valve rather than repairing it. What is an advantage of valvuloplasty that the nurse should cite? A) The procedure can be performed on an outpatient basis in a physicians office. B) Repaired valves tend to function longer than replaced valves. C) The procedure is not associated with a risk for infection. D) Lower doses of antirejection drugs are required than with valve replacement.

B Feedback: In general, valves that undergo valvuloplasty function longer than prosthetic valve replacements and patients do not require continuous anticoagulation. Valvuloplasty carries a risk of infection, like all surgical procedures, and it is not performed in a physicians office. Antirejection drugs are unnecessary because foreign tissue is not introduced. 9. The nurse is reviewing the echocardiography results

During a CPR class, a participant asks about the difference between cardioversion and defibrillation. What would be the instructors best response? A) Cardioversion is done on a beating heart; defibrillation is not. B) The difference is the timing of the delivery of the electric current. C) Defibrillation is synchronized with the electrical activity of the heart, but cardioversion is not. D) Cardioversion is always attempted before defibrillation because it has fewer risks.

B Feedback: One major difference between cardioversion and defibrillation is the timing of the delivery of electrical current. In cardioversion, the delivery of the electrical current is synchronized with the patients electrical events; in defibrillation, the delivery of the current is immediate and unsynchronized. Both can be done on beating heart (i.e., in a dysrhythmia). Cardioversion is not necessarily attempted first.

The nurse is creating a plan of care for a patient with acute coronary syndrome. What nursing action should be included in the patients care plan? A) Facilitate daily arterial blood gas (ABG) sampling. B) Administer supplementary oxygen, as needed. C) Have patient maintain supine positioning when in bed. D) Perform chest physiotherapy, as indicated.

B Feedback: Oxygen should be administered along with medication therapy to assist with symptom relief. Administration of oxygen raises the circulating level of oxygen to reduce pain associated with low levels of myocardial oxygen. Physical rest in bed with the head of the bed elevated or in a supportive chair helps decrease chest discomfort and dyspnea. ABGs are diagnostic, not therapeutic, and they are rarely needed on a daily basis. Chest physiotherapy is not used in the treatment of ACS.

The nurse is providing care for a patient who has just been diagnosed with peripheral arterial occlusive disease (PAD). What assessment finding is most consistent with this diagnosis? A) Numbness and tingling in the distal extremities B) Unequal peripheral pulses between extremities C) Visible clubbing of the fingers and toes D) Reddened extremities with muscle atrophy

B Feedback: PAD assessment may manifest as unequal pulses between extremities, with the affected leg cooler and paler than the unaffected leg. Intermittent claudication is far more common than sensations of numbness and tingling. Clubbing and muscle atrophy are not associated with PAD.

The critical care nurse is caring for a patient who has had an MI. The nurse should expect to assist with establishing what hemodynamic monitoring procedure to assess the patients left ventricular function? A) Central venous pressure (CVP) monitoring B) Pulmonary artery pressure monitoring (PAPM) C) Systemic arterial pressure monitoring (SAPM) D) Arterial blood gases (ABG)

B Feedback: PAPM is used to assess left ventricular function. CVP is used to assess right ventricular function; SAPM is used for continual assessment of BP. ABG are used to assess for acidic and alkalotic levels in the blood.

During an adult patients last two office visits, the nurse obtained BP readings of 122/84 mm Hg and 130/88 mm Hg, respectively. How would this patients BP be categorized? A) Normal B) Prehypertensive C) Stage 1 hypertensive D) Stage 2 hypertensive

B Feedback: Prehypertension is defined systolic BP of 120 to 139 mm Hg or diastolic BP of 80 to 89 mm Hg.

While auscultating a patients heart sounds, the nurse hears an extra heart sound immediately after the second heart sound (S2). An audible S3 would be considered an expected finding in what patient? A) An older adult B) A 20-year-old patient C) A patient who has undergone valve replacement D) A patient who takes a beta-adrenergic blocker

B Feedback: S3 represents a normal finding in children and adults up to 35 or 40 years of age. In these cases, it is called a physiologic S3. It is an abnormal finding in a patient with an artificial valve, an older adult, or a patient who takes a beta blocker.

The nurse is analyzing a rhythm strip. What component of the ECG corresponds to the resting state of the patients heart? A) P wave B) T wave C) U wave D) QRS complex

B Feedback: The T wave specifically represents ventricular muscle depolarization, also referred to as the resting state. Ventricular muscle depolarization does not result in the P wave, U wave, or QRS complex.

New nurses on the telemetry unit have been paired with preceptors. One new nurse asks her preceptor to explain depolarization. What would be the best answer by the preceptor? A) Depolarization is the mechanical contraction of the heart muscles. B) Depolarization is the electrical stimulation of the heart muscles. C) Depolarization is the electrical relaxation of the heart muscles. D) Depolarization is the mechanical relaxation of the heart muscles.

B Feedback: The electrical stimulation of the heart is called depolarization, and the mechanical contraction is called systole. Electrical relaxation is called repolarization, and mechanical relaxation is called diastole.

During a shift assessment, the nurse is identifying the clients point of maximum impulse (PMI). Where will the nurse best palpate the PMI? A) Left midclavicular line of the chest at the level of the nipple B) Left midclavicular line of the chest at the fifth intercostal space C) Midline between the xiphoid process and the left nipple D) Two to three centimeters to the left of the sternum

B Feedback: The left ventricle is responsible for the apical beat or the point of maximum impulse, which is normally palpated in the left midclavicular line of the chest wall at the fifth intercostal space.

A brain (B-type) natriuretic peptide (BNP) sample has been drawn from an older adult patient who has been experienced vital fatigue and shortness of breath. This test will allow the care team to investigate the possibility of what diagnosis? A) Pleurisy B) Heart failure C) Valve dysfunction D) Cardiomyopathy

B Feedback: The level of BNP in the blood increases as the ventricular walls expand from increased pressure, making it a helpful diagnostic, monitoring, and prognostic tool in the setting of HF. It is not specific to cardiomyopathy, pleurisy, or valve dysfunction.

The nurse is caring for a patient with refractory atrial fibrillation who underwent the maze procedure several months ago. The nurse reviews the result of the patients most recent cardiac imaging, which notes the presence of scarring on the atria. How should the nurse best respond to this finding? A) Recognize that the procedure was unsuccessful. B) Recognize this as a therapeutic goal of the procedure. C) Liaise with the care team in preparation for repeating the maze procedure. D) Prepare the patient for pacemaker implantation.

B Feedback: The maze procedure is an open heart surgical procedure for refractory atrial fibrillation. Small transmural incisions are made throughout the atria. The resulting formation of scar tissue prevents reentry conduction of the electrical impulse. Consequently, scar formation would constitute a successful procedure. There is no indication for repeating the procedure or implanting a pacemaker.

A patient with pericarditis has just been admitted to the CCU. The nurse planning the patients care should prioritize what nursing diagnosis? A) Anxiety related to pericarditis B) Acute pain related to pericarditis C) Ineffective tissue perfusion related to pericarditis D) Ineffective breathing pattern related to pericarditis

B Feedback: The most characteristic symptom of pericarditis is chest pain, although pain also may be located beneath the clavicle, in the neck, or in the left trapezius (scapula) region. The pain or discomfort usually remains fairly constant, but it may worsen with deep inspiration and when lying down or turning. Anxiety is highly plausible and should be addressed, but chest pain is a nearly certain accompaniment to the disease. Breathing and tissue perfusion are likely to be at risk, but pain is certain, especially in the early stages of treatment.

When assessing a patient diagnosed with angina pectoris it is most important for the nurse to gather what information? A) The patients activities limitations and level of consciousness after the attacks B) The patients symptoms and the activities that precipitate attacks C) The patients understanding of the pathology of angina D) The patients coping strategies surrounding the attacks

B Feedback: The nurse must gather information about the patients symptoms and activities, especially those that precede and precipitate attacks of angina pectoris. The patients coping, understanding of the disease, and status following attacks are all important to know, but causative factors are a primary focus of the assessment interview.

The nurse is caring for a 68-year-old patient the nurse suspects has digoxin toxicity. In addition to physical assessment, the nurse should collect what assessment datum? A) Skin turgor B) Potassium level C) White blood cell count D) Peripheral pulses

B Feedback: The serum potassium level is monitored because the effect of digoxin is enhanced in the presence of hypokalemia and digoxin toxicity may occur. Skin turgor, white cell levels, and peripheral pulses are not normally affected in cases of digitalis toxicity.

In preparation for cardiac surgery, a patient was taught about measures to prevent venous thromboembolism. What statement indicates that the patient clearly understood this education? A) Ill try to stay in bed for the first few days to allow myself to heal. B) Ill make sure that I dont cross my legs when Im resting in bed. C) Ill keep pillows under my knees to help my blood circulate better. D) Ill put on those compression stockings if I get pain in my calves.

B Feedback: To prevent venous thromboembolism, patients should avoid crossing the legs. Activity is generally begun as soon as possible and pillows should not be placed under the popliteal space. Compression stockings are often used to prevent venous thromboembolism, but they would not be applied when symptoms emerge.

The physical therapist notifies the nurse that a patient with coronary artery disease (CAD) experiences a much greater-than-average increase in heart rate during physical therapy. The nurse recognizes that an increase in heart rate in a patient with CAD may result in what? A) Development of an atrial-septal defect B) Myocardial ischemia C) Formation of a pulmonary embolism D) Release of potassium ions from cardiac cells

B Feedback: Unlike other arteries, the coronary arteries are perfused during diastole. An increase in heart rate shortens diastole and can decrease myocardial perfusion. Patients, particularly those with CAD, can develop myocardial ischemia. An increase in heart rate will not usually result in a pulmonary embolism or create electrolyte imbalances. Atrial-septal defects are congenital.

The staff educator is teaching a CPR class. Which of the following aspects of defibrillation should the educator stress to the class? A) Apply the paddles directly to the patients skin. B) Use a conducting medium between the paddles and the skin. C) Always use a petroleum-based gel between the paddles and the skin. D) Any available liquid can be used between the paddles and the skin.

B Feedback: Use multifunction conductor pads or paddles with a conducting medium between the paddles and the skin (the conducting medium is available as a sheet, gel, or paste). Do not use gels or pastes with poor electrical conductivity.

The nurse is caring for a patient who is admitted to the medical unit for the treatment of a venous ulcer in the area of her lateral malleolus that has been unresponsive to treatment. What is the nurse most likely to find during an assessment of this patients wound? A) Hemorrhage B) Heavy exudate C) Deep wound bed D) Pale-colored wound bed

B Feedback: Venous ulcerations in the area of the medial or lateral malleolus (gaiter area) are typically large, superficial, and highly exudative. Venous hypertension causes extravasation of blood, which discolors the area of the wound bed. Bleeding is not normally present.

A patient with mitral valve stenosis is receiving health education at an outpatient clinic. To minimize the patients symptoms, the nurse should teach the patient to do which of the following? A) Eat a high-protein, low-carbohydrate diet. B) Avoid activities that cause an increased heart rate. C) Avoid large crowds and public events. D) Perform deep breathing and coughing exercises.

B Patients with mitral stenosis are advised to avoid strenuous activities, competitive sports, and pregnancy, all of which increase heart rate. Infection prevention is important, but avoiding crowds is not usually necessary. Deep breathing and coughing are not likely to prevent exacerbations of symptoms and increased protein intake is not necessary.

An adult patient is admitted to the ED with chest pain. The patient states that he had developed unrelieved chest pain that was present for approximately 20 minutes before coming to the hospital. To minimize cardiac damage, the nurse should expect to administer which of the following interventions? A) Thrombolytics, oxygen administration, and nonsteroidal anti-inflammatories B) Morphine sulphate, oxygen, and bed rest C) Oxygen and beta-adrenergic blockers D) Bed rest, albuterol nebulizer treatments, and oxygen

B The patient with suspected MI should immediately receive supplemental oxygen, aspirin, nitroglycerin, and morphine. Morphine sulphate reduces preload and decreases workload of the heart, along with increased oxygen from oxygen therapy and bed rest. With decreased cardiac demand, this provides the best chance of decreasing cardiac damage. NSAIDs and beta-blockers are not normally indicated. Albuterol, which is a medication used to manage asthma and respiratory conditions, will increase the heart rate.

A patient converts from normal sinus rhythm at 80 bpm to atrial fibrillation with a ventricular response at 166 bpm. Blood pressure is 162/74 mm Hg. Respiratory rate is 20 breaths per minute with normal chest expansion and clear lungs bilaterally. IV heparin and Cardizem are given. The nurse caring for the patient understands that the main goal of treatment is what? A) Decrease SA node conduction B) Control ventricular heart rate C) Improve oxygenation D) Maintain anticoagulation

B Treatment for atrial fibrillation is to terminate the rhythm or to control ventricular rate. This is a priority because it directly affects cardiac output. A rapid ventricular response reduces the time for ventricular filling, resulting in a smaller stroke volume. Control of rhythm is the initial treatment of choice, followed by anticoagulation with heparin and then Coumadin.

The nurse is reviewing the medication administration record of a patient who takes a variety of medications for the treatment of hypertension. What potential therapeutic benefits of antihypertensives should the nurse identify? Select all that apply. A) Increased venous return B) Decreased peripheral resistance C) Decreased blood volume D) Decreased strength and rate of myocardial contractions E) Decreased blood viscosity

B, C, D Feedback: The medications used for treating hypertension decrease peripheral resistance, blood volume, or the strength and rate of myocardial contraction. Antihypertensive medications do not increase venous return or decrease blood viscosity.

The nurse is relating the deficits in a patients synchronization of the atrial and ventricular events to his diagnosis. What are the physiologic characteristics of the nodal and Purkinje cells that provide this synchronization? Select all that apply. A) Loop connectivity B) Excitability C) Automaticity D) Conductivity E) Independence

B, C, D Feedback: Three physiologic characteristics of two types of specialized electrical cells, the nodal cells and the Purkinje cells, provide this synchronization: automaticity, or the ability to initiate an electrical impulse; excitability, or the ability to respond to an electrical impulse; and conductivity, the ability to transmit an electrical impulse from one cell to another. Loop connectivity is a distracter for this question. Independence of the cells has nothing to do with the synchronization described in the scenario.

A patient is admitted to the cardiac care unit for an electrophysiology (EP) study. What goal should guide the planning and execution of the patients care? A) Ablate the area causing the dysrhythmia. B) Freeze hypersensitive cells. C) Diagnose the dysrhythmia. D) Determine the nursing plan of care.

C Feedback: A patient may undergo an EP study in which electrodes are placed inside the heart to obtain an intracardiac ECG. This is used not only to diagnose the dysrhythmia but also to determine the most effective treatment plan. However, because an EP study is invasive, it is performed in the hospital and may require that the patient be admitted.

A nurse is working with a patient who has been scheduled for a percutaneous coronary intervention (PCI) later in the week. What anticipatory guidance should the nurse provide to the patient? A) He will remain on bed rest for 48 to 72 hours after the procedure. B) He will be given vitamin K infusions to prevent bleeding following PCI. C) A sheath will be placed over the insertion site after the procedure is finished. D) The procedure will likely be repeated in 6 to 8 weeks to ensure success.

C Feedback: A sheath is placed over the PCI access site and kept in place until adequate coagulation is achieved. Patients resume activity a few hours after PCI and repeated treatments may or may not be necessary. Anticoagulants, not vitamin K, are administered during PCI.

The nurse is caring for a patient who has just had an implantable cardioverter defibrillator (ICD) placed. What is the priority area for the nurses assessment? A) Assessing the patients activity level B) Facilitating transthoracic echocardiography C) Vigilant monitoring of the patients ECG D) Close monitoring of the patients peripheral perfusion

C Feedback: After a permanent electronic device (pacemaker or ICD) is inserted, the patients heart rate and rhythm are monitored by ECG. This is a priority over peripheral circulation and activity. Echocardiography is not indicated.

A lipid profile has been ordered for a patient who has been experiencing cardiac symptoms. When should a lipid profile be drawn in order to maximize the accuracy of results? A) As close to the end of the day as possible B) After a meal high in fat C) After a 12-hour fast D) Thirty minutes after a normal meal

C Feedback: Although cholesterol levels remain relatively constant over 24 hours, the blood specimen for the lipid profile should be obtained after a 12-hour fast.

Diagnostic imaging reveals that the quantity of fluid in a clients pericardial sac is dangerously increased. The nurse should collaborate with the other members of the care team to prevent the development of what complication? A) Pulmonary edema B) Pericardiocentesis C) Cardiac tamponade D) Pericarditis

C Feedback: An increase in pericardial fluid raises the pressure within the pericardial sac and compresses the heart, eventually causing cardiac tamponade. Pericardiocentesis is the treatment for this complication. Pericarditis and pulmonary edema do not result from this pathophysiological process.

A 79-year-old man is admitted to the medical unit with digital gangrene. The man states that his problems first began when he stubbed his toe going to the bathroom in the dark. In addition to this trauma, the nurse should suspect that the patient has a history of what health problem? A) Raynauds phenomenon B) CAD C) Arterial insufficiency D) Varicose veins

C Feedback: Arterial insufficiency may result in gangrene of the toe (digital gangrene), which usually is caused by trauma. The toe is stubbed and then turns black. Raynauds, CAD and varicose veins are not the usual causes of digital gangrene in the elderly.

The nurse notes that a patient has developed a cough productive for mucoid sputum, is short of breath, has cyanotic hands, and has noisy, moist-sounding, rapid breathing. These symptoms and signs are suggestive of what health problem? A) Pericarditis B) Cardiomyopathy C) Pulmonary edema D) Right ventricular hypertrophy

C Feedback: As a result of decreased cerebral oxygenation, the patient with pulmonary edema becomes increasingly restless and anxious. Along with a sudden onset of breathlessness and a sense of suffocation, the patients hands become cold and moist, the nail beds become cyanotic (bluish), and the skin turns ashen (gray). The pulse is weak and rapid, and the neck veins are distended. Incessant coughing may occur, producing increasing quantities of foamy sputum. Pericarditis, ventricular hypertrophy, and cardiomyopathy do not involve wet breath sounds or mucus production.

The triage nurse in the ED is assessing a patient with chronic HF who has presented with worsening symptoms. In reviewing the patients medical history, what is a potential primary cause of the patients heart failure? A) Endocarditis B) Pleural effusion C) Atherosclerosis D) Atrial-septal defect

C Feedback: Atherosclerosis of the coronary arteries is the primary cause of HF. Pleural effusion, endocarditis, and an atrial-septal defect are not health problems that contribute to the etiology of HF.

The nurse is caring for a patient who has developed obvious signs of pulmonary edema. What is the priority nursing action? A) Lay the patient flat. B) Notify the family of the patients critical state. C) Stay with the patient. D) Update the physician.

C Feedback: Because the patient has an unstable condition, the nurse must remain with the patient. The physician must be updated promptly, but the patient should not be left alone in order for this to happen. Supine positioning is unlikely to relieve dyspnea. The family should be informed, but this is not the priority action. 37. A cardiac patients

The triage nurse in the ED is assessing a patient who has presented with complaint of pain and swelling in her right lower leg. The patients pain became much worse last night and appeared along with fever, chills, and sweating. The patient states, I hit my leg on the car door 4 or 5 days ago and it has been sore ever since. The patient has a history of chronic venous insufficiency. What intervention should the nurse anticipate for this patient? A) Platelet transfusion to treat thrombocytopenia B) Warfarin to treat arterial insufficiency C) Antibiotics to treat cellulitis D) Heparin IV to treat VTE

C Feedback: Cellulitis is the most common infectious cause of limb swelling. The signs and symptoms include acute onset of swelling, localized redness, and pain; it is frequently associated with systemic signs of fever, chills, and sweating. The patient may be able to identify a trauma that accounts for the source of infection. Thrombocytopenia is a loss or decrease in platelets and increases a patients risk of bleeding; this problem would not cause these symptoms. Arterial insufficiency would present with ongoing pain related to activity. This patient does not have signs and symptoms of VTE.

A patient is recovering in the hospital from cardiac surgery. The nurse has identified the diagnosis of risk for ineffective airway clearance related to pulmonary secretions. What intervention best addresses this risk? A) Administration of bronchodilators by nebulizer B) Administration of inhaled corticosteroids by metered dose inhaler (MDI) C) Patients consistent performance of deep breathing and coughing exercises D) Patients active participation in the cardiac rehabilitation program

C Feedback: Clearance of pulmonary secretions is accomplished by frequent repositioning of the patient, suctioning, and chest physical therapy, as well as educating and encouraging the patient to breathe deeply and cough. Medications are not normally used to achieve this goal. Rehabilitation is important, but will not necessarily aid the mobilization of respiratory secretions.

The nurse is assessing a patient new to the clinic. Records brought to the clinic with the patient show the patient has hypertension and that her current BP readings approximate the readings from when she was first diagnosed. What contributing factor should the nurse first explore in an effort to identify the cause of the clients inadequate BP control? A) Progressive target organ damage B) Possibility of medication interactions C) Lack of adherence to prescribed drug therapy D) Possible heavy alcohol use or use of recreational drugs

C Feedback: Deviation from the therapeutic program is a significant problem for people with hypertension and other chronic conditions requiring lifetime management. An estimated 50% of patients discontinue their medications within 1 year of beginning to take them. Consequently, this is a more likely problem than substance use, organ damage, or adverse drug interactions.

A patient with hypertrophic cardiomyopathy (HCM) has been admitted to the medical unit. During the nurses admission interview, the patient states that she takes over-the-counter water pills on a regular basis. How should the nurse best respond to the fact that the patient has been taking diuretics? A) Encourage the patient to drink at least 2 liters of fluid daily. B) Increase the patients oral sodium intake. C) Inform the care provider because diuretics are contraindicated. D) Ensure that the patients fluid balance is monitored vigilantly.

C Feedback: Diuretics are contraindicated in patients with HCM, so the primary care provider should be made aware. Adjusting the patients sodium or fluid intake or fluid monitoring does not address this important contraindication.

A nurse is preparing a patient for scheduled transesophageal echocardiography. What action should the nurse perform? Test Bank - Brunner & Suddarth's Textbook of Medical-Surgical Nursing 14e (Hinkle 2017) 497 A) Instruct the patient to drink 1 liter of water before the test. B) Administer IV benzodiazepines and opioids. C) Inform the patient that she will remain on bed rest following the procedure. D) Inform the patient that an access line will be initiated in her femoral artery.

C Feedback: During the recovery period, the patient must maintain bed rest with the head of the bed elevated to 45 degrees. The patient must be NPO 6 hours preprocedure. The patient is sedated to make him or her comfortable, but will not be heavily sedated, and opioids are not necessary. Also, the patient will have a peripheral IV line initiated preprocedure.

The nurse is providing care for a patient with a diagnosis of hypertension. The nurse should consequently assess the patient for signs and symptoms of which other health problem? A) Migraines B) Atrial-septal defect C) Atherosclerosis D) Thrombocytopenia

C Feedback: Hypertension is both a sign and a risk factor for atherosclerotic heart disease. It is not associated with structural cardiac defects, low platelet levels, or migraines.

The nurse is assessing a patient who was admitted to the critical care unit 3 hours ago following cardiac surgery. The nurses most recent assessment reveals that the patients left pedal pulses are not palpable and that the right pedal pulses are rated at +2. What is the nurses best response? A) Document this expected assessment finding during the initial postoperative period. B) Reposition the patient with his left leg in a dependent position. C) Inform the patients physician of this assessment finding. D) Administer an ordered dose of subcutaneous heparin.

C Feedback: If a pulse is absent in any extremity, the cause may be prior catheterization of that extremity, chronic peripheral vascular disease, or a thromboembolic obstruction. The nurse immediately reports newly identified absence of any pulse.

The nurse is caring for an acutely ill patient who is on anticoagulant therapy. The patient has a comorbidity of renal insufficiency. How will this patients renal status affect heparin therapy? A) Heparin is contraindicated in the treatment of this patient. B) Heparin may be administered subcutaneously, but not IV. C) Lower doses of heparin are required for this patient. D) Coumadin will be substituted for heparin.

C Feedback: If renal insufficiency exists, lower doses of heparin are required. Coumadin cannot be safely and effectively used as a substitute and there is no contraindication for IV administration.

A patient with mitral stenosis exhibits new symptoms of a dysrhythmia. Based on the pathophysiology of this disease process, the nurse would expect the patient to exhibit what heart rhythm? A) Ventricular fibrillation (VF) B) Ventricular tachycardia (VT) C) Atrial fibrillation D) Sinus bradycardia

C Feedback: In patients with mitral valve stenosis, the pulse is weak and often irregular because of atrial fibrillation. Bradycardia, VF, and VT are not characteristic of this valvular disorder.

An adult patient with third-degree AV block is admitted to the cardiac care unit and placed on continuous cardiac monitoring. What rhythm characteristic will the ECG most likely show? A) PP interval and RR interval are irregular. B) PP interval is equal to RR interval. C) Fewer QRS complexes than P waves D) PR interval is constant.

C Feedback: In third-degree AV block, no atrial impulse is conducted through the AV node into the ventricles. As a result, there are impulses stimulating the atria and impulses stimulating the ventricles. Therefore, there are more P waves than QRS complexes due to the difference in the natural pacemaker (nodes) rates of the heart. The other listed ECG changes are not consistent with this diagnosis.

The nurse is caring for an adult patient who had symptoms of unstable angina upon admission to the hospital. What nursing diagnosis underlies the discomfort associated with angina? A) Ineffective breathing pattern related to decreased cardiac output B) Anxiety related to fear of death C) Ineffective cardiopulmonary tissue perfusion related to coronary artery disease (CAD) D) Impaired skin integrity related to CAD

C Feedback: Ineffective cardiopulmonary tissue perfusion directly results in the symptoms of discomfort associated with angina. Anxiety and ineffective breathing may result from angina chest pain, but they are not the causes. Skin integrity is not impaired by the effects of angina.

A patient in a hypertensive emergency is admitted to the ICU. The nurse anticipates that the patient will be treated with IV vasodilators, and that the primary goal of treatment is what? A) Lower the BP to reduce onset of neurologic symptoms, such as headache and vision changes. B) Decrease the BP to a normal level based on the patients age. C) Decrease the mean arterial pressure between 20% and 25% in the first hour of treatment. D) Reduce the BP to 120/75 mm Hg as quickly as possible.

C Feedback: Initially, the treatment goal in hypertensive emergencies is to reduce the mean arterial pressure by 25% in the first hour of treatment, with further reduction over the next 24 hours. Lowering the BP too fast may cause hypotension in a patient whose body has adjusted to hypertension and could cause a stroke, MI, or visual changes. Neurologic symptoms should be addressed, but this is not the primary focus of treatment planning.

A nurse on a medical unit is caring for a patient who has been diagnosed with lymphangitis. When reviewing this patients medication administration record, the nurse should anticipate which of the following? A) Coumadin (warfarin) B) Lasix (furosemide) C) An antibiotic D) An antiplatelet aggregator

C Feedback: Lymphangitis is an acute inflammation of the lymphatic channels caused by an infectious process. Antibiotics are always a component of treatment. Diuretics are of nominal use. Anticoagulants and antiplatelet aggregators are not indicated in this form of infection.

A patient has been admitted with an aortic valve stenosis and has been scheduled for a balloon valvuloplasty in the cardiac catheterization lab later today. During the admission assessment, the patient tells the nurse he has thoracolumbar scoliosis and is concerned about lying down for any extended period of time. What is a priority action for the nurse? A) Arrange for an alternative bed. B) Measure the degree of the curvature. C) Notify the surgeon immediately. D) Note the scoliosis on the intake assessment.

C Feedback: Most often used for mitral and aortic valve stenosis, balloon valvuloplasty is contraindicated for patients with left atrial or ventricular thrombus, severe aortic root dilation, significant mitral valve regurgitation, thoracolumbar scoliosis, rotation of the great vessels, and other cardiac conditions that require open heart surgery. Therefore notifying the physician would be the priority over further physical assessment. An alternative bed would be unnecessary and documentation is not a sufficient response.

A medical nurse has admitted four patients over the course of a 12-hour shift. For which patient would assessment of ankle-brachial index (ABI) be most clearly warranted? A) A patient who has peripheral edema secondary to chronic heart failure B) An older adult patient who has a diagnosis of unstable angina C) A patient with poorly controlled type 1 diabetes who is a smoker D) A patient who has community-acquired pneumonia and a history of COPD

C Feedback: Nurses should perform a baseline ABI on any patient with decreased pulses or any patient 50 years of age or older with a history of diabetes or smoking. The other answers do not apply.

A patient who is postoperative day 1 following a CABG has produced 20 mL of urine in the past 3 hours and the nurse has confirmed the patency of the urinary catheter. What is the nurses most appropriate action? A) Document the patients low urine output and monitor closely for the next several hours. B) Contact the dietitian and suggest the need for increased oral fluid intake. C) Contact the patients physician and suggest assessment of fluid balance and renal function. D) Increase the infusion rate of the patients IV fluid to prompt an increase in renal function.

C Feedback: Nursing management includes accurate measurement of urine output. An output of less than 1 mL/kg/h may indicate hypovolemia or renal insufficiency. Prompt referral is necessary. IV fluid replacement may be indicated, but is beyond the independent scope of the dietitian or nurse.

A nurse is closely monitoring a patient who has recently been diagnosed with an abdominal aortic aneurysm. What assessment finding would signal an impending rupture of the patients aneurysm? A) Sudden increase in blood pressure and a decrease in heart rate B) Cessation of pulsating in an aneurysm that has previously been pulsating visibly C) Sudden onset of severe back or abdominal pain D) New onset of hemoptysis

C Feedback: Signs of impending rupture include severe back or abdominal pain, which may be persistent or intermittent. Impending rupture is not typically signaled by increased blood pressure, bradycardia, cessation of pulsing, or hemoptysis.

A patient has undergone diagnostic testing and received a diagnosis of sinus bradycardia attributable to sinus node dysfunction. When planning this patients care, what nursing diagnosis is most appropriate? A) Acute pain B) Risk for unilateral neglect C) Risk for activity intolerance D) Risk for fluid volume excess

C Feedback: Sinus bradycardia causes decreased cardiac output that is likely to cause activity intolerance. It does not typically cause pain, fluid imbalances, or neglect of a unilateral nature.

A student nurse is taking care of an elderly patient with hypertension during a clinical experience. The instructor asks the student about the relationships between BP and age. What would be the best answer by the student? A) Because of reduced smooth muscle tone in blood vessels, blood pressure tends to go down with age, not up. B) Decreases in the strength of arteries and the presence of venous insufficiency cause hypertension in the elderly. C) Structural and functional changes in the cardiovascular system that occur with age contribute to increases in blood pressure. D) The neurologic system of older adults is less efficient at monitoring and regulating blood pressure.

C Feedback: Structural and functional changes in the heart and blood vessels contribute to increases in BP that occur with aging. Venous insufficiency does not cause hypertension, however. Increased BP is not primarily a result of neurologic changes.

The nurse is preparing to administer warfarin (Coumadin) to a client with deep vein thrombophlebitis (DVT). Which laboratory value would most clearly indicate that the patients warfarin is at therapeutic levels? A) Partial thromboplastin time (PTT) within normal reference range B) Prothrombin time (PT) eight to ten times the control C) International normalized ratio (INR) between 2 and 3 D) Hematocrit of 32%

C Feedback: The INR is most often used to determine if warfarin is at a therapeutic level; an INR of 2 to 3 is considered therapeutic. Warfarin is also considered to be at therapeutic levels when the clients PT is 1.5 to 2 times the control. Higher values indicate increased risk of bleeding and hemorrhage, whereas lower values indicate increased risk of blood clot formation. Heparin, not warfarin, prolongs PTT. Hematocrit does not provide information on the effectiveness of warfarin; however, a falling hematocrit in a client taking warfarin may be a sign of hemorrhage.

The nursing educator is presenting a case study of an adult patient who has abnormal ventricular depolarization. This pathologic change would be most evident in what component of the ECG? A) P wave B) T wave C) QRS complex D) U wave

C Feedback: The QRS complex represents the depolarization of the ventricles and, as such, the electrical activity of that ventricle.

The nurse is admitting a patient with complaints of dyspnea on exertion and fatigue. The patients ECG shows dysrhythmias that are sometimes associated with left ventricular hypertrophy. What diagnostic tool would be most helpful in diagnosing cardiomyopathy? A) Cardiac catheterization B) Arterial blood gases C) Echocardiogram D) Exercise stress test

C Feedback: The echocardiogram is one of the most helpful diagnostic tools because the structure and function of the ventricles can be observed easily. The ECG is also important, and can demonstrate dysrhythmias and changes consistent with left ventricular hypertrophy. Cardiac catheterization specifically addresses coronary artery function and arterial blood gases evaluate gas exchange and acid balance. Stress testing is not normally used to differentiate cardiomyopathy from other cardiac pathologies.

A patient is undergoing diagnostic testing for mitral stenosis. What statement by the patient during the nurses interview is most suggestive of this valvular disorder? A) I get chest pain from time to time, but it usually resolves when I rest. B) Sometimes when Im resting, I can feel my heart skip a beat. C) Whenever I do any form of exercise I get terribly short of breath. D) My feet and ankles have gotten terribly puffy the last few weeks.

C Feedback: The first symptom of mitral stenosis is often breathing difficulty (dyspnea) on exertion as a result of pulmonary venous hypertension. Patients with mitral stenosis are likely to show progressive fatigue as a result of low cardiac output. Palpitations occur in some patients, but dyspnea is a characteristic early symptom. Peripheral edema and chest pain are atypical.

The critical care nurse is caring for a patient just admitted in a hypertensive emergency. The nurse should anticipate the administration of what medication? A) Warfarin (Coumadin) B) Furosemide (Lasix) C) Sodium nitroprusside (Nitropress) D) Ramipril (Altace)

C Feedback: The medications of choice in hypertensive emergencies are those that have an immediate effect. IV vasodilators, including sodium nitroprusside (Nitropress), nicardipine hydrochloride (Cardene), clevidipine (Cleviprex), fenoldopam mesylate (Corlopam), enalaprilat, and nitroglycerin, have immediate actions that are short lived (minutes to 4 hours), and they are therefore used for initial treatment. Ramipril is administered orally and would not meet the patients immediate need for BP management. Diuretics, such as Lasix, are not used as initial treatments and there is no indication for anticoagulants such as Coumadin.

The nurse is caring for a recent immigrant who has been diagnosed with mitral valve regurgitation. The nurse should know that in developing countries the most common cause of mitral valve regurgitation is what? A) A decrease in gamma globulins B) An insect bite C) Rheumatic heart disease and its sequelae D) Sepsis and its sequelae

C Feedback: The most common cause of mitral valve regurgitation in developing countries is rheumatic heart disease and its sequelae.

A patient has had a myocardial infarction and has been diagnosed as having damage to the layer of the heart responsible for the pumping action. You are aware that the damage occurred where? A) Endocardium B) Pericardium C) Myocardium D) Visceral pericardium

C Feedback: The myocardium is the layer of the heart responsible for the pumping action.

The ED nurse is caring for a patient with a suspected MI. What drug should the nurse anticipate administering to this patient? A) Oxycodone B) Warfarin C) Morphine D) Acetaminophen

C Feedback: The patient with suspected MI is given aspirin, nitroglycerin, morphine, an IV beta- blocker, and other medications, as indicated, while the diagnosis is being confirmed. Tylenol, warfarin, and oxycodone are not typically used.

A patient presents to the clinic complaining of the inability to grasp objects with her right hand. The patients right arm is cool and has a difference in blood pressure of more than 20 mm Hg compared with her left arm. The nurse should expect that the primary care provider may diagnose the woman with what health problem? A) Lymphedema B) Raynauds phenomenon C) Upper extremity arterial occlusive disease D) Upper extremity VTE

C Feedback: The patient with upper extremity arterial occlusive disease typically complains of arm fatigue and pain with exercise (forearm claudication) and inability to hold or grasp objects (e.g., combing hair, placing objects on shelves above the head) and, occasionally, difficulty driving. Assessment findings include coolness and pallor of the affected extremity, decreased capillary refill, and a difference in arm blood pressures of more than 20 mm Hg. These symptoms are not closely associated with Raynauds or lymphedema. The upper extremities are rare sites for VTE.

The nurse is caring for a patient with acute pericarditis. What nursing management should be instituted to minimize complications? A) The nurse keeps the patient isolated to prevent nosocomial infections. B) The nurse encourages coughing and deep breathing. C) The nurse helps the patient with activities until the pain and fever subside. D) The nurse encourages increased fluid intake until the infection resolves.

C Feedback: To minimize complications, the nurse helps the patient with activity restrictions until the pain and fever subside. As the patients condition improves, the nurse encourages gradual increases of activity. Actions to minimize complications of acute pericarditis do not include keeping the patient isolated. Due to pain, coughing and deep breathing are not normally encouraged. An increase in fluid intake is not always necessary.

The nurse is caring for a patient admitted with unstable angina. The laboratory result for the initial troponin I is elevated in this patient. The nurse should recognize what implication of this assessment finding? A) This is only an accurate indicator of myocardial damage when it reaches its peak in 24 hours. B) Because the patient has a history of unstable angina, this is a poor indicator of myocardial injury. C) This is an accurate indicator of myocardial injury. D) This result indicates muscle injury, but does not specify the source.

C Feedback: Troponin I, which is specific to cardiac muscle, is elevated within hours after myocardial injury. Even with a diagnosis of unstable angina, this is an accurate indicator of myocardial injury.

A nurse is performing blood pressure screenings at a local health fair. While obtaining subjective assessment data from a patient with hypertension, the nurse learns that the patient has a family history of hypertension and she herself has high cholesterol and lipid levels. The patient says she smokes one pack of cigarettes daily and drinks about a pack of beer every day. The nurse notes what nonmodifiable risk factor for hypertension? A) Hyperlipidemia B) Excessive alcohol intake C) A family history of hypertension D) Closer adherence to medical regimen

C Feedback: Unlike cholesterol levels, alcohol intake and adherence to treatment, family history is not modifiable.

The nurse is participating in the care conference for a patient with ACS. What goal should guide the care teams selection of assessments, interventions, and treatments? A) Maximizing cardiac output while minimizing heart rate B) Decreasing energy expenditure of the myocardium C) Balancing myocardial oxygen supply with demand D) Increasing the size of the myocardial muscle

C Feedback: Balancing myocardial oxygen supply with demand (e.g., as evidenced by the relief of chest pain) is the top priority in the care of the patient with ACS. Treatment is not aimed directly at minimizing heart rate because some patients experience bradycardia. Increasing the size of the myocardium is never a goal. Reducing the myocardiums energy expenditure is often beneficial, but this must be balanced with productivity.

An older adult patient has been treated for a venous ulcer and a plan is in place to prevent the occurrence of future ulcers. What should the nurse include in this plan? A) Use of supplementary oxygen to aid tissue oxygenation B) Daily use of normal saline compresses on the lower limbs C) Daily administration of prophylactic antibiotics D) A high-protein diet that is rich in vitamins

D Feedback: A diet that is high in protein, vitamins C and A, iron, and zinc is encouraged to promote healing and prevent future ulcers. Prophylactic antibiotics and saline compresses are not used to prevent ulcers. Oxygen supplementation does not prevent ulcer formation.

The nurse has entered a patients room and found the patient unresponsive and not breathing. What is the nurses next appropriate action? A) Palpate the patients carotid pulse. B) Illuminate the patients call light. C) Begin performing chest compressions. D) Activate the Emergency Response System (ERS).

D Feedback: After checking for responsiveness and breathing, the nurse should activate the ERS. Assessment of carotid pulse should follow and chest compressions may be indicated. Illuminating the call light is an insufficient response.

The critical care nurse is caring for a patient who is receiving cyclosporine postoperative heart transplant. The patient asks the nurse to remind him what this medication is for. How should the nurse best respond? A) Azathioprine decreases the risk of thrombus formation. B) Azathioprine ensures adequate cardiac output. C) Azathioprine increases the number of white blood cells. D) Azathioprine minimizes rejection of the transplant.

D Feedback: After heart transplant, patients are constantly balancing the risk of rejection with the risk of infection. Most commonly, patients receive cyclosporine or tacrolimus (FK506, Prograf), azathioprine (Imuran), or mycophenolate mofetil (CellCept), and corticosteroids (prednisone) to minimize rejection. Cyclosporine does not prevent thrombus formation, enhance cardiac output, or increase white cell counts.

A patient is a candidate for percutaneous balloon valvuloplasty, but is concerned about how this procedure will affect her busy work schedule. What guidance should the nurse provide to the patient? A) Patients generally stay in the hospital for 6 to 8 days. B) Patients are kept in the hospital until they are independent with all aspects of their care. C) Patients need to stay in the hospital until they regain normal heart function for their age. D) Patients usually remain at the hospital for 24 to 48 hours.

D Feedback: After undergoing percutaneous balloon valvuloplasty, the patient usually remains in the hospital for 24 to 48 hours. Prediagnosis levels of heart function are not always attainable and the patient does not need to be wholly independent prior to discharge.

An ECG has been ordered for a newly admitted patient. What should the nurse do prior to electrode placement? A) Clean the skin with providone-iodine solution. B) Ensure that the area for electrode placement is dry. C) Apply tincture of benzoin to the electrode sites and wait for it to become tacky. D) Gently abrade the skin by rubbing the electrode sites with dry gauze or cloth.

D Feedback: An ECG is obtained by slightly abrading the skin with a clean dry gauze pad and placing electrodes on the body at specific areas. The abrading of skin will enhance signal transmission. Disinfecting the skin is unnecessary and conduction gel is used.

Graduated compression stockings have been prescribed to treat a patients venous insufficiency. What education should the nurse prioritize when introducing this intervention to the patient? A) The need to take anticoagulants concurrent with using compression stockings B) The need to wear the stockings on a one day on, one day off schedule C) The importance of wearing the stockings around the clock to ensure maximum benefit D) The importance of ensuring the stockings are applied evenly with no pressure points

D Feedback: Any type of stocking can inadvertently become a tourniquet if applied incorrectly (i.e., rolled tightly at the top). In such instances, the stockings produce rather than prevent stasis. For ambulatory patients, graduated compression stockings are removed at night and reapplied before the legs are lowered from the bed to the floor in the morning. They are used daily, not on alternating days. Anticoagulants are not always indicated in patients who are using compression stockings.

The triage nurse in the ED is performing a rapid assessment of a man with complaints of severe chest pain and shortness of breath. The patient is diaphoretic, pale, and weak. When the patient collapses, what should the nurse do first? A) Check for a carotid pulse. B) Apply supplemental oxygen. C) Give two full breaths. D) Gently shake and shout, Are you OK?

D Feedback: Assessing responsiveness is the first step in basic life support. Opening the airway and checking for respirations should occur next. If breathing is absent, two breaths should be given, usually accompanied by supplementary oxygen. Circulation is checked by palpating the carotid artery.

The nurse is caring for a patient with right ventricular hypertrophy and consequently decreased right ventricular function. What valvular disorder may have contributed to this patients diagnosis? A) Mitral valve regurgitation B) Aortic stenosis C) Aortic regurgitation D) Mitral valve stenosis

D Feedback: Because no valve protects the pulmonary veins from the backward flow of blood from the atrium, the pulmonary circulation becomes congested. As a result, the right ventricle must contract against an abnormally high pulmonary arterial pressure and is subjected to excessive strain. Eventually, the right ventricle fails. None of the other listed valvular disorders has this pathophysiological effect.

The cardiac monitor alarm alerts the critical care nurse that the patient is showing no cardiac rhythm on the monitor. The nurses rapid assessment suggests cardiac arrest. In providing cardiac resuscitation documentation, how will the nurse describe this initial absence of cardiac rhythm? A) Pulseless electrical activity (PEA) B) Ventricular fibrillation C) Ventricular tachycardia D) Asystole

D Feedback: Cardiac arrest occurs when the heart ceases to produce an effective pulse and circulate blood. It may be caused by a cardiac electrical event such as ventricular fibrillation, ventricular tachycardia, profound bradycardia, or when there is no heart rhythm at all (asystole). Cardiac arrest may also occur when electrical activity is present, but there is ineffective cardiac contraction or circulating volume, which is PEA. Asystole is the only condition that involves the absolute absence of a heart rhythm.

The OR nurse is explaining to a patient that cardiac surgery requires the absence of blood from the surgical field. At the same time, it is imperative to maintain perfusion of body organs and tissues. What technique for achieving these simultaneous goals should the nurse describe? A) Coronary artery bypass graft (CABG) B) Percutaneous transluminal coronary angioplasty (PTCA) C) Atherectomy D) Cardiopulmonary bypass

D Feedback: Cardiopulmonary bypass is often used to circulate and oxygenate blood mechanically while bypassing the heart and lungs. PTCA, atherectomy, and CABG are all surgical procedures, none of which achieves the two goals listed.

The nurse is caring for an 82-year-old patient. The nurse knows that changes in cardiac structure and function occur in older adults. What is a normal change expected in the aging heart of an older adult? Test Bank - Brunner & Suddarth's Textbook of Medical-Surgical Nursing 14e (Hinkle 2017) 494 A) Decreased left ventricular ejection time B) Decreased connective tissue in the SA and AV nodes and bundle branches C) Thinning and flaccidity of the cardiac values D) Widening of the aorta

D Feedback: Changes in cardiac structure and function are clearly observable in the aging heart. Aging results in decreased elasticity and widening of the aorta, thickening and rigidity of the cardiac valves, increased connective tissue in the SA and AV nodes and bundle branches, and an increased left ventricular ejection time (prolonged systole).

A nurse is providing health education to a patient scheduled for cryoablation therapy. The nurse should describe what aspect of this treatment? A) Peeling away the area of endocardium responsible for the dysrhythmia B) Using electrical shocks directly to the endocarduim to eliminate the source of dysrhythmia C) Using high-frequency sound waves to eliminate the source of dysrhythmia D) Using a cooled probe to eliminate the source of dysrhythmia

D Feedback: Cryoablation therapy involves using a cooled probe to create a small scar on the endocardium to eliminate the source of the dysrhythmias. Endocardium resection involves peeling away a specified area of the endocardium. Electrical ablation involves using shocks to eliminate the area causing the dysrhythmias. Radio frequency ablation uses high-frequency sound waves to destroy the area causing the dysrhythmias

The public health nurse is participating in a health fair and interviews a patient with a history of hypertension, who is currently smoking one pack of cigarettes per day. She denies any of the most common manifestations of CAD. Based on these data, the nurse would expect the focuses of CAD treatment most likely to be which of the following? A) Drug therapy and smoking cessation B) Diet and drug therapyC) Diet therapy only D) Diet therapy and smoking cessation

D Feedback: Due to the absence of symptoms, dietary therapy would likely be selected as the first-line treatment for possible CAD. Drug therapy would be determined based on a number of considerations and diagnostics findings, but would not be directly indicated. Smoking cessation is always indicated, regardless of the presence or absence of symptoms.

The nurse is assessing an older adult patient with numerous health problems. What assessment datum indicates an increase in the patients risk for heart failure (HF)? A) The patient takes Lasix (furosemide) 20 mg/day. B) The patients potassium level is 4.7 mEq/L. C) The patient is an African American man. D) The patients age is greater than 65.

D Feedback: HF is the most common reason for hospitalization of people older than 65 years of age and is the second most common reason for visits to a physicians office. A potassium level of 4.7 mEq/L is within reference range and does not indicate an increased risk for HF. The fact that the patient takes Lasix 20 mg/day does not indicate an increased risk for HF, although this drug is often used in the treatment of HF. The patient being an African American man does not indicate an increased risk for HF.

A community health nurse teaching a group of adults about preventing and treating hypertension. The nurse should encourage these participants to collaborate with their primary care providers and regularly monitor which of the following? A) Heart rate B) Sodium levels C) Potassium levels D) Blood lipid levels

D Feedback: Hypertension often accompanies other risk factors for atherosclerotic heart disease, such as dyslipidemia (abnormal blood fat levels), obesity, diabetes, metabolic syndrome, and a sedentary lifestyle. Individuals with hypertension need to monitor their sodium intake, but hypernatremia is not a risk factor for hypertension. In many patients, heart rate does not correlate closely with BP. Potassium levels do not normally relate to BP.

A 55-year-old patient comes to the clinic for a routine check-up. The patients BP is 159/100 mm Hg and the physician diagnoses hypertension after referring to previous readings. The patient asks why it is important to treat hypertension. What would be the nurses best response? A) Hypertension can cause you to develop dangerous blood clots in your legs that can migrate to your lungs. B) Hypertension puts you at increased risk of type 1 diabetes and cancer in your age group. C) Hypertension is the leading cause of death in people your age. D) Hypertension greatly increases your risk of stroke and heart disease.

D Feedback: Hypertension, particularly elevated systolic BP, increases the risk of death, stroke, and heart failure in people older than 50 years. Hypertension is not a direct precursor to pulmonary emboli, and it does not put older adults at increased risk of type 1 diabetes or cancer. It is not the leading cause of death in people 55 years of age.

A patient in hypertensive emergency is being cared for in the ICU. The patient has become hypovolemic secondary to natriuresis. What is the nurses most appropriate action? A) Add sodium to the patients IV fluid, as ordered. B) Administer a vasoconstrictor, as ordered. C) Promptly cease antihypertensive therapy. D) Administer normal saline IV, as ordered.

D Feedback: If there is volume depletion secondary to natriuresis caused by the elevated BP, then volume replacement with normal saline can prevent large, sudden drops in BP when antihypertensive medications are administered. Sodium administration, cessation of antihypertensive therapy, and administration of vasoconstrictors are not normally indicated.

The nurse is caring for a patient on telemetry. The patients ECG shows a shortened PR interval, slurring of the initial QRS deflection, and prolonged QRS duration. What does this ECG show? A) Sinus bradycardia B) Myocardial infarction C) Lupus-like syndrome D) Wolf-Parkinson-White (WPW) syndrome

D Feedback: In WPW syndrome there is a shortened PR interval, slurring (called a delta wave) of the initial QRS deflection, and prolonged QRS duration. These characteristics are not typical of the other listed cardiac anomalies.

A patient in hypertensive urgency is admitted to the hospital. The nurse should be aware of what goal of treatment for a patient in hypertensive urgency? A) Normalizing BP within 2 hours B) Obtaining a BP of less than 110/70 mm Hg within 36 hours C) Obtaining a BP of less than 120/80 mm Hg within 36 hours D) Normalizing BP within 24 to 48 hours

D Feedback: In cases of hypertensive urgency, oral agents can be administered with the goal of normalizing BP within 24 to 48 hours. For patients with this health problem, a BP of 120/80 mm Hg may be unrealistic.

A nurse working in a long-term care facility is performing the admission assessment of a newly admitted, 85-year-old resident. During inspection of the residents feet, the nurse notes that she appears to have early evidence of gangrene on one of her great toes. The nurse knows that gangrene in the elderly is often the first sign of what? A) Chronic venous insufficiency B) Raynauds phenomenon C) VTE D) PAD

D Feedback: In elderly people, symptoms of PAD may be more pronounced than in younger people. In elderly patients who are inactive, gangrene may be the first sign of disease. Venous insufficiency does not normally manifest with gangrene. Similarly, VTE and Raynauds phenomenon do not cause the ischemia that underlies gangrene.

A patient presents to the walk-in clinic complaining of intermittent chest pain on exertion, which is eventually attributed to angina. The nurse should inform the patient that angina is most often attributable to what cause? A) Decreased cardiac output B) Decreased cardiac contractility C) Infarction of the myocardium D) Coronary arteriosclerosis

D Feedback: In most cases, angina pectoris is due to arteriosclerosis. The disease is not a result of impaired cardiac output or contractility. Infarction may result from untreated angina, but it is not a cause of the disease.

A patient calls his cardiologists office and talks to the nurse. He is concerned because he feels he is being defibrillated too often. The nurse tells the patient to come to the office to be evaluated because the nurse knows that the most frequent complication of ICD therapy is what? A) Infection B) Failure to capture C) Premature battery depletion D) Oversensing of dysrhythmias

D Feedback: Inappropriate delivery of ICD therapy, usually due to oversensing of atrial and sinus tachycardias with a rapid ventricular rate response, is the most frequent complication of ICD. Infections, failure to capture, and premature battery failure are less common.

The nurse is screening a number of adults for hypertension. What range of blood pressure is considered normal? A) Less than 140/90 mm Hg B) Less than 130/90 mm Hg C) Less than 129/89 mm Hg D) Less than 120/80 mm Hg

D Feedback: JNC 7 defines a blood pressure of less than 120/80 mm Hg as normal, 120 to 129/80 to 89 mm Hg as prehypertension, and 140/90 mm Hg or higher as hypertension.

A patient comes to the walk-in clinic complaining of frequent headaches. While assessing the patients vital signs, the nurse notes the BP is 161/101 mm Hg. According to JNC 7, how would this patients BP be defined if a similar reading were obtained at a subsequent office visit? A) High normal B) Normal C) Stage 1 hypertensive D) Stage 2 hypertensive

D Feedback: JNC 7 defines stage 2 hypertension as a reading 160/100 mm Hg.

A patients recently elevated BP has prompted the primary care provider to prescribe furosemide (Lasix). The nurse should closely monitor which of the following? A) The clients oxygen saturation level B) The patients red blood cells, hematocrit, and hemoglobin C) The patients level of consciousness D) The patients potassium level

D Feedback: Loop diuretics can cause potassium depletion. They do not normally affect level of consciousness, erythrocytes, or oxygen saturation.

A critical care nurse is caring for a patient with a pulmonary artery catheter in place. What does this catheter measure that is particularly important in critically ill patients? A) Pulmonary artery systolic pressure B) Right ventricular afterload C) Pulmonary artery pressure D) Left ventricular preload

D Feedback: Monitoring of the pulmonary artery diastolic and pulmonary artery wedge pressures is particularly important in critically ill patients because it is used to evaluate left ventricular filling pressures (i.e., left ventricular preload). This device does not directly measure the other listed aspects of cardiac function.

Preoperative education is an important part of the nursing care of patients having coronary artery revascularization. When explaining the pre- and postoperative regimens, the nurse would be sure to include education about which subject? A) Symptoms of hypovolemia B) Symptoms of low blood pressure C) Complications requiring graft removal D) Intubation and mechanical ventilation

D Feedback: Most patients remain intubated and on mechanical ventilation for several hours after surgery. It is important that patients realize that this will prevent them from talking, and the nurse should reassure them that the staff will be able to assist them with other means of communication. Teaching would generally not include symptoms of low blood pressure or hypovolemia, as these are not applicable to most patients. Teaching would also generally not include rare complications that would require graft removal.

The nurse is providing care to a patient who has just undergone an electrophysiologic (EP) study. The patient states that she is nervous about things going wrong during the procedure. What is the nurses best response? A) This is basically a risk-free procedure. B) Thousands of patients undergo EP every year. C) Remember that this is a step that will bring you closer to enjoying good health. D) The whole team will be monitoring you very closely for the entire procedure.

D Feedback: Patients who are to undergo an EP study may be anxious about the procedure and its outcome. A detailed discussion involving the patient, the family, and the electrophysiologist usually occurs to ensure that the patient can give informed consent and to reduce the patients anxiety about the procedure. It is inaccurate to state that EP is risk-free and stating that it is common does not necessarily relieve the patients anxiety. Characterizing EP as a step toward good health does not directly address the patients anxiety.

A group of nurses are participating in orientation to a telemetry unit. What should the staff educator tell this class about ST segments? A) They are the part of an ECG that reflects systole. B) They are the part of an ECG used to calculate ventricular rate and rhythm. C) They are the part of an ECG that reflects the time from ventricular depolarization through repolarization. D) They are the part of an ECG that represents early ventricular repolarization.

D Feedback: ST segment is the part of an ECG that reflects the end of the QRS complex to the beginning of the T wave. The part of an ECG that reflects repolarization of the ventricles is the T wave. The part of an ECG used to calculate ventricular rate and rhythm is the RR interval. The part of an ECG that reflects the time from ventricular depolarization through repolarization is the QT interval.

The nurse is caring for a patient who is undergoing an exercise stress test. Prior to reaching the target heart rate, the patient develops chest pain. What is the nurses most appropriate response? A) Administer sublingual nitroglycerin to allow the patient to finish the test. B) Initiate cardiopulmonary resuscitation. C) Administer analgesia and slow the test. D) Stop the test and monitor the patient closely.

D Feedback: Signs of myocardial ischemia would necessitate stopping the test. CPR would only be necessary if signs of cardiac or respiratory arrest were evident.

When assessing the patient with pericardial effusion, the nurse will assess for pulsus paradoxus. Pulsus paradoxus is characterized by what assessment finding? A) A diastolic blood pressure that is lower during exhalation B) A diastolic blood pressure that is higher during inhalation C) A systolic blood pressure that is higher during exhalation D) A systolic blood pressure that is lower during inhalation

D Feedback: Systolic blood pressure that is markedly lower during inhalation is called pulsus paradoxus. The difference in systolic pressure between the point that is heard during exhalation and the point that is heard during inhalation is measured. Pulsus paradoxus exceeding 10 mm Hg is abnormal.

A nurse is reviewing the physiological factors that affect a patients cardiovascular health and tissue oxygenation. What is the systemic arteriovenous oxygen difference? A) The average amount of oxygen removed by each organ in the body B) The amount of oxygen removed from the blood by the heart C) The amount of oxygen returning to the lungs via the pulmonary artery D) The amount of oxygen in aortic blood minus the amount of oxygen in the vena caval blood

D Feedback: The average amount of oxygen removed collectively by all of the body tissues is about 25%. This means that the blood in the vena cava contains about 25% less oxygen than aortic blood. This is known as the systemic arteriovenous oxygen difference. The other answers do not apply.

An OR nurse is preparing to assist with a coronary artery bypass graft (CABG). The OR nurse knows that the vessel most commonly used as source for a CABG is what? A) Brachial artery B) Brachial vein C) Femoral artery D) Greater saphenous vein

D Feedback: The greater saphenous vein is the most commonly used graft site for CABG. The right and left internal mammary arteries, radial arteries, and gastroepiploic artery are other graft sites used, though not as frequently. The femoral artery, brachial artery, and brachial vein are never harvested.

A patient who has recently recovered from a systemic viral infection is undergoing diagnostic testing for myocarditis. Which of the nurses assessment findings is most consistent with myocarditis? A) Sudden changes in level of consciousness (LOC) B) Peripheral edema and pulmonary edema C) Pleuritic chest pain D) Flulike symptoms

D Feedback: The most common symptoms of myocarditis are flulike. Chest pain, edema, and changes in LOC are not characteristic of myocarditis.

Following cardiac resuscitation, a patient has been placed in a state of mild hypothermia before being transferred to the cardiac intensive care unit. The nurses assessment reveals that the patient is experiencing neuromuscular paralysis. How should the nurse best respond? A) Administer hypertonic IV solution. B) Administer a bolus of warned normal saline. C) Reassess the patient in 15 minutes. D) Document this as an expected assessment finding.

D Feedback: The nurse caring for a patient with hypothermia (passive or induced) needs to monitor for appropriate level of cooling, sedation, and neuromuscular paralysis to prevent seizures; myoclonus; and shivering. Neuromuscular paralysis is an expected finding and does not necessitate further interventions.

A community health nurse is planning an educational campaign addressing hypertension. The nurse should anticipate that the incidence and prevalence of hypertension are likely to be highest among members of what ethnic group? A) Pacific Islanders B) African Americans C) Asian-Americans D) Hispanics

D Feedback: The prevalence of uncontrolled hypertension varies by ethnicity, with Hispanics and African Americans having the highest prevalence at approximately 63% and 57%, respectively.

A critically ill patient is admitted to the ICU. The physician decides to use intra-arterial pressure monitoring. After this intervention is performed, what assessment should the nurse prioritize in the plan of care? A) Fluctuations in core body temperature B) Signs and symptoms of esophageal varices C) Signs and symptoms of compartment syndrome D) Perfusion distal to the insertion site

D Feedback: The radial artery is the usual site selected. However, placement of a catheter into the radial artery can further impede perfusion to an area that has poor circulation. As a result, the tissue distal to the cannulated artery can become ischemic or necrotic. Vigilant assessment is thus necessary. Alterations in temperature and the development of esophageal varices or compartment syndrome are not high risks.

The nurse is teaching a patient diagnosed with aortic stenosis appropriate strategies for attempting to relieve the symptom of angina without drugs. What should the nurse teach the patient? A) To eat a small meal before taking nitroglycerin B) To drink a glass of milk before taking nitroglycerin C) To engage in 15 minutes of light exercise before taking nitroglycerin D) To rest and relax before taking nitroglycerin

D Feedback: The venous dilation that results from nitroglycerin decreases blood return to the heart, thus decreasing cardiac output and increasing the risk of syncope and decreased coronary artery blood flow. The nurse teaches the patient about the importance of attempting to relieve the symptoms of angina with rest and relaxation before taking nitroglycerin and to anticipate the potential adverse effects. Exercising, eating, and drinking are not recommended prior to using nitroglycerin.

The home health nurse is caring for a patient who has a comorbidity of hypertension. What assessment question most directly addresses the possibility of worsening hypertension? A) Are you eating less salt in your diet? B) How is your energy level these days? C) Do you ever get chest pain when you exercise? D) Do you ever see spots in front of your eyes?

D Feedback: To identify complications or worsening hypertension, the patient is questioned about blurred vision, spots in front of the eyes, and diminished visual acuity. The heart, nervous system, and kidneys are also carefully assessed, but angina pain and decreased energy are not normally suggestive of worsening hypertension. Sodium limitation is a beneficial lifestyle modification, but nonadherence to this is not

A patient is brought into the ED by family members who tell the nurse the patient grabbed his chest and complained of substernal chest pain. The care team recognizes the need to monitor the patients cardiac function closely while interventions are performed. What form of monitoring should the nurse anticipate? A) Left-sided heart catheterization B) Cardiac telemetry C) Transesophageal echocardiography D) Hardwire continuous ECG monitoring

D Feedback: Two types of continuous ECG monitoring techniques are used in health care settings: hardwire cardiac monitoring, found in EDs, critical care units, and progressive care units; and telemetry, found in general nursing care units or outpatient cardiac rehabilitation programs. Cardiac catheterization and transesophageal echocardiography would not be used in emergent situations to monitor cardiac function.

A nurse in the rehabilitation unit is caring for an older adult patient who is in cardiac rehabilitation following an MI. The nurses plan of care calls for the patient to walk for 10 minutes 3 times a day. The patient questions the relationship between walking and heart function. How should the nurse best reply? A) The arteries in your legs constrict when you walk and allow the blood to move faster and with more pressure on the tissue. B) Walking increases your heart rate and blood pressure. Therefore your heart is under less stress. C) Walking helps your heart adjust to your new arteries and helps build your self-esteem. D) When you walk, the muscles in your legs contract and pump the blood in your veins back toward your heart, which allows more blood to return to your heart.

D Feedback: Veins, unlike arteries, are equipped with valves that allow blood to move against the force of gravity. The legs have one-way bicuspid valves that prevent blood from seeping backward as it moves forward by the muscles in our legs pressing on the veins as we walk and increasing venous return. Leg arteries do constrict when walking, which allows the blood to move faster and with more pressure on the tissue, but the greater concern is increasing the flow of venous blood to the heart. Walking increases, not decreases, the heart pumping ability, which increases heart rate and blood pressure and the hearts ability to manage stress. Walking does help the heart adjust to new arteries and may enhance self-esteem, but the patient had an MIthere are no new arteries.

The nurse is caring for patient who tells the nurse that he has an angina attack beginning. What is the nurses most appropriate initial action? A) Have the patient sit down and put his head between his knees. B) Have the patient perform pursed-lip breathing. C) Have the patient stand still and bend over at the waist. D) Place the patient on bed rest in a semi-Fowlers position.

D Feedback: When a patient experiences angina, the patient is directed to stop all activities and sit or rest in bed in a semi-Fowlers position to reduce the oxygen requirements of the ischemic myocardium. Pursed-lip breathing and standing will not reduce workload to the same extent. No need to have the patient put his head between his legs because cerebral perfusion is not lacking.

A nurse is assessing a new patient who is diagnosed with PAD. The nurse cannot feel the pulse in the patients left foot. How should the nurse proceed with assessment? A) Have the primary care provider order a CT. B) Apply a tourniquet for 3 to 5 minutes and then reassess. C) Elevate the extremity and attempt to palpate the pulses. D) Use Doppler ultrasound to identify the pulses.

D Feedback: When pulses cannot be reliably palpated, a hand-held continuous wave (CW) Doppler ultrasound device may be used to hear (insonate) the blood flow in vessels. CT is not normally warranted and the application of a tourniquet poses health risks and will not aid assessment. Elevating the extremity would make palpation more difficult.

An ED nurse is assessing an adult woman for a suspected MI. When planning the assessment, the nurse should be cognizant of what signs and symptoms of MI that are particularly common in female patients? Select all that apply. A) Shortness of breath B) Chest pain C) Anxiety D) Numbness E) Weakness

D, E Feedback: Although these symptoms are not wholly absent in men, many women have been found to have atypical symptoms of MI, including indigestion, nausea, palpitations, and numbness. Shortness of breath, chest pain, and anxiety are common symptoms of MI among patients of all ages and genders.


Kaugnay na mga set ng pag-aaral

Fundamental of nursing 1 midterm

View Set

Systems Analysis and Design Ch 1, 2, 3, 4, 5, 6

View Set

Pathophysiology Reproductive Questions

View Set

Computer Architecture and Fetch-execute cycle

View Set

Chapter 8: Attitudes and Persuasive Communications

View Set

Greetings - 1 (2)- 你好,朋友/老师/同学! (pinyin - English)

View Set